You are on page 1of 61

English Medium

UPSC CSE Free Material (OPTIMISTIC IAS)

VISION IAS

TEST-29 TEST BOOKLET


Test Booklet Series

C
GENERAL STUDIES (P) 20 21 – Test–320 6
Time Allowed: Two Hours Maximum Marks: 200

FULL LENGTH TEST-5


(COMPLETE SYLLABUS):
Indian Constitution,
Political System and Govern
ance + Geography + History of India + I
Development + Ecology & Environment + General Science
ndian Economy & Social

+ Current Affairs
UPSC CSE Free Material (OPTIMISTIC IAS)
DO NOT OPEN THIS BOOKLET UNTIL YOU ARE ASKED TO DO SO
1 www.visionias.in ©Vision IAS
DEEPAK PHOTOSTAT 9310521834
UPSC CSE Free Material (OPTIMISTIC IAS)
1. The Montagu Declaration of 1917: 4. Consider the following statements regarding
1. recommended that self-government 'bioceramics':
would be granted to India immediately
1. Bioceramics are materials specially
after the first world war.
2. led to the introduction of the system of developed to replace soft organ tissue in
direct elections and universal adult the human body.
franchise in India for the first time.
2. All bioceramics are biologically inert
Select the correct answer using the code
and do not react with other tissues.
given below.
(a) 1 only 3. Alumina and zirconia are used in the
(b) 2 only production of bioceramics.
(c) Both 1 and 2
Which of the statements given above are
(d) Neither 1 nor 2
correct?

2. Consider the following statements: (a) 1 and 2 only


1. Translation is the process of conversion (b) 2 and 3 only
of a sequence of an RNA molecule to a
(c) 3 only
sequence of DNA.
2. Transcription is the process of synthesis (d) 1, 2 and 3
of RNA from the DNA.
Which of the statements given above is/are
5. With reference to vaccines, consider the
correct?
following statements:
(a) 1 only
(b) 2 only 1. Inactivated vaccines use the killed
(c) Both 1 and 2 version of the germ that causes disease
(d) Neither 1 nor 2
while live-attenuated vaccines use a

3. Consider the following statements with weakened form of the germ that causes a

respect to the Jury Act of 1827: disease.


1. It provided that Christians were 2. Inactivated vaccines create a stronger
exempted from being tried either by
and long-lasting immune response
Hindu and Muslim juror.
2. Raja Ram Mohan Roy condemned the compared to live-attenuated vaccines.
act and sent petitions to British Which of the statements given above is/are
Parliament opposing the act. correct?
Which of the statements given above is/are
(a) 1 only
correct?
(a) 1 only (b) 2 only
(b) 2 only (c) Both 1 and 2
(c) Both 1 and 2
(d) Neither 1 nor 2
(d) Neither 1 nor 2

2 www.visionias.in ©Vision IAS

DEEPAK PHOTOSTAT 9310521834


6. Consider the following statements: 8. With reference to Chaitanya Mahaprabhu,
1. The festival is celebrated to mark the consider the following statements:

change of color of spring waters by 1. Chaitanya and Shankaradeva were

Goddess Ragnya Devi in the month of contemporaries.

May/June every year. 2. He popularised the Krishna cult in the

2. The changing color of the spring water Bengal region.

every year during the festival was cited 3. He denounced the Brahmanical authority

as a miracle by Swami Vivekanand. and advocated the abolition of the caste

3. The festival is one of the most important system.

festivals in the Union Territory of Which of the statements given above is/are

Jammu and Kashmir. correct?

Which of the following Indian festival is (a) 2 only

characterized by the above-given (b) 1 and 2 only

statements? (c) 1 and 3 only

(a) Jude Sheetal (d) 1, 2 and 3

(b) Ambubachi Mela


(c) Mela Ksheer Bhavani 9. Which of the following statements best

(d) Herath describes the Doctrine of Proportionality?


(a) It envisages that a public authority

7. Consider the following statements regarding should maintain a sense of proportion

Sambhar lake in Rajasthan: between his particular goals and the

1. It is surrounded by Aravalli hills on all means he employs to achieve those

sides. goals.

2. No microorganisms are found in this (b) It envisages that the state must maintain

lake because of its high salt content. proportion in the treatment of all citizens

3. Nearly half of India's salt production as equal in the eyes of the law.

comes from this lake. (c) It suggests that the state must provide for

Which of the statements given above is/are welfare of its citizens in accordance with

correct? their existing economic position.

(a) 1 only (d) It suggests that the carbon footprint of

(b) 1 and 2 only the present generation must be

(c) 2 and 3 only proportionate for the future generations

(d) 1, 2 and 3 to mitigate successfully.

3 www.visionias.in ©Vision IAS

DEEPAK PHOTOSTAT 9310521834


10. With reference to the Ninth schedule of the 13. Recently, India has signed Strategic
Indian constitution, consider the following Partnership with which of the following
statements:
International Agencies?
1. It was the first schedule that was added
after the commencement of the (a) International Atomic Energy Agency
constitution. (b) Food and Agriculture Organization
2. It was enacted to protect laws related to (c) International Labour Organization
agrarian reforms and for abolishing the
(d) International Energy Agency
zamindari system.
3. The acts and regulations specified in the
ninth Schedule are not subject to judicial 14. The term “Laschamps Excursion” is
review at all. sometimes mentioned in the news in the
Which of the statements given above is/are
context of the discussion of
correct?
(a) 1 and 2 only (a) the gradual disappearance of
(b) 1 only temperature difference below the
(c) 1, 2 and 3 oceanic and continental crusts at great
(d) 2 and 3 only
depths.

11. Consider the following statements regarding (b) short duration illumination of comets by
Global Mangrove Alliance: the reflection of Sun light during its
1. It is an initiative of the United Nations journey through the Solar system.
Environment Programme (UNEP).
(c) the rare linear planetary alignment of
2. It aims to increase the global area of
mangrove habitat by 20 per cent over the Mars, Mercury, Venus, Jupiter, Saturn,
current extent by 2030. and Moon expected to occur in 2040.
Which of the statements given above is/are (d) short reversal of the Earth’s magnetic
correct?
field during the end of the Last Glacial
(a) 1 only
(b) 2 only Period.
(c) Both 1 and 2
(d) Neither 1 nor 2 15. Arrange the following India-Nepal boundary
locations from West to East:
12. Exfoliation tors are small to big size
boulders which are smooth and rounded 1. Lipulekh
formed due to 2. Kalapani
(a) Gravitational forces such as overburden 3. Limpiyadhura
pressure, load and shearing stress.
Select the correct answer using the code
(b) Removal of overlying rock load because
of continued erosion. given below.
(c) Differential heating and resulting (a) 1-3-2
expansion and contraction of surface (b) 2-3-1
layers.
(c) 3-2-1
(d) River’s vertical and lateral corrasion in
its mountain course. (d) 2-1-3

4 www.visionias.in ©Vision IAS

DEEPAK PHOTOSTAT 9310521834


16. The word 'Seneca Guns' is sometimes 18. Consider the following statements with
mentioned in media in reference to respect to Mauryan rule:
1. Megasthenes visited the Mauryan
(a) nanotechnology based radioactive
empire during the reign of Chandragupta
ablation therapy. Maurya.
(b) the phenomenon of loud noises similar 2. The second Buddhist Council was held
during the reign of Ashoka.
to that of thunder often heard in the
3. The provincial capital 'Suvarnagiri'
coastal areas. mentioned in the rock edicts, was
(c) noiseless geological underwater located in the Northern province of the
eruptions in the Pacific Ring of Fire. empire.
4. Sanchi Stupa was built during the reign
(d) an all-weather mobile air defense
of Ashoka.
system. Which of the statements given above are
correct?
(a) 2 and 3 only
17. With reference to the collegium system for
(b) 1 and 4 only
the appointment of judges to the Supreme (c) 1, 2 and 3 only
Court, consider the following statements: (d) 1, 3 and 4 only
1. The collegium for the selection of a
19. In which of the following coastal regions of
Supreme Court judge comprises of the
India, the Kaipad system of rice cultivation
Chief Justice of India along with two is practised?
senior-most judges of the Supreme (a) Konkan coast
(b) Malabar coast
Court.
(c) Coramandal coast
2. In case of lack of consensus within the
(d) Gujarat coast
collegium system, the Chief Justice of
India can exercise veto power. 20. With reference to the 'Tree Cities of the
World' programme, consider the following
3. Once a recommendation is approved by
statements:
the collegium, the Ministry of Law and 1. To be recognized as a tree city of the
Justice is required to approve or send world, the forest area of the city should
be at least 33 per cent of its geographical
back for reconsideration within thirty
area.
days.
2. It is one of the initiatives of the Food
Which of the statements given above is/are and Agriculture Organisation (FAO).
correct? Which of the statements given above is/are
correct?
(a) 2 only
(a) 1 only
(b) 2 and 3 only (b) 2 only
(c) 1 and 3 only (c) Both 1 and 2

(d) None (d) Neither 1 nor 2


5 www.visionias.in ©Vision IAS

DEEPAK PHOTOSTAT 9310521834


21. Consider the following statements: 24. Consider the following statements with
1. A Progressive tax is a tax in which tax reference to Consumer Welfare Fund:
rate increases as the taxable amount 1. Its objective is to provide financial
increases.
assistance to promote and protect the
2. Inflation acts as a progressive tax in an
welfare of the consumers.
economy.
Which of the statements given above is/are 2. It was set up under the Consumer
correct? Protection Act, 2019.
(a) 1 only 3. It is being operated under the Ministry of
(b) 2 only Consumer Affairs, Food & Public
(c) Both 1 and 2 Distribution.
(d) Neither 1 nor 2
Which of the statements given above are
correct?
22. This protected area having semi-evergreen to
dry deciduous vegetation is a part of the (a) 1 and 2 only
Mayurbhanj Elephant Reserve. It had been (b) 2 and 3 only
included in the World Network of Biosphere (c) 1 and 3 only
Reserves by UNESCO. Many forest- (d) 1, 2 and 3
dwelling communities celebrate the Akhand
Shikar ritual which is considered to be one
25. With reference to Landholders' Society
of the reasons for the recent forest fires.
founded in 1837-38, consider the following
Which protected area is being described in
the above passage? statements:
(a) Kanha National Park 1. It was a political association prominent
(b) Similipal Tiger Reserve in the Bengal region.
(c) Jim Corbett National Park 2. It aimed to strengthen the vested
(d) Nagarhole Tiger Reserve interests of Zamindars.
3. It was later merged with the British
23. In the recently published Human
Indian Association of Bengal.
Development Report, 2020, United Nations
Development Programme (UNDP) has Which of the statements given above is/are
released an experimental metric Planetary correct?
Pressures-Adjusted HDI (PHDI). Which of (a) 1 and 2 only
the following correctly describes PHDI? (b) 2 and 3 only
(a) It is the relative level of human (c) 1 only
development in countries lying in high
(d) 1, 2 and 3
and low atmospheric pressure belts.
(b) It is the level of human development
adjusted by per capita material footprint 26. Galwan Valley, which has been a flashpoint
and carbon dioxide emissions. between China and India, lies in which of
(c) It is the opportunity loss of human the following mountain ranges?
development caused by socio-economic (a) Ladakh Range
hardships during COVID-led slowdown. (b) Karakoram Range
(d) It is the level of human development
(c) Greater Himalayas
adjusted by losses due to natural
(d) Zanskar Range
disasters and calamities.
6 www.visionias.in ©Vision IAS

DEEPAK PHOTOSTAT 9310521834


27. Recently, the UN High Commissioner for 29. With reference to the British rule in India,
Human Rights has raised concern over consider the following statements regarding
misuse of the sedition law in India. In this the outcomes of the Shimla conference
context, consider the following statements (Wavell plan) 1945:
regarding 'Sedition' under section 124A of 1. All the members of the Viceroy's
the Indian Penal Code: Executive Council would be Indians.
1. 'Sedition' is also mentioned as one of the 2. Only the defense, would remain in
reasonable restrictions under Article British hands until the power was
19(2) of the Indian Constitution. ultimately transferred to Indians.
2. A person charged under this law is 3. Muslim league boycotted the
barred from a government job.
conference.
3. Bal Gangadhar Tilak was the first person
Which of the statements given above is/are
to be charged under sedition during the
correct?
freedom movement.
(a) 1 and 2 only
Which of the statements given above is/are
(b) 2 only
correct?
(c) 1 and 3 only
(a) 1 and 2 only
(d) 1, 2 and 3
(b) 2 only
(c) 2 and 3 only
30. With reference to the Panchayat Elections,
(d) None
consider the following statements:
1. General Elections to the three tiers of
28. With reference to the administration by
Panchayats are held simultaneously.
Ashtapradhans during Shivaji period,
2. Elections to grama panchayats must be
Consider the following pairs:
held on the basis of non-political party
Title Duty
symbols.
1. Sumanta : Master of ceremonies
2. Amatya : Finance and general 3. Model code of conduct for Panchayat

administration elections is issued by respective State

3. Waqenavis : Religious administration Election Commissions.

Which of the pairs given above is/are Which of the statements given above is/are

correct? correct?

(a) 1 only (a) 1 and 2 only

(b) 2 only (b) 2 only

(c) 2 and 3 only (c) 1 and 3 only


(d) 1, 2 and 3 (d) 1, 2 and 3
7 www.visionias.in ©Vision IAS

DEEPAK PHOTOSTAT 9310521834


31. Consider the following statements as factors 34. Renouncement of practices derogatory to the
responsible for the establishment of the dignity of women is mentioned in which of
Home Rule League: the following provisions of the Indian
1. High taxation and wartime miseries
Constitution?
motivated people to participate in an
aggressive movement. (a) Fundamental Rights
2. The Congress wanted to set up an (b) Directive Principles of State Policy
extended arm for a proactive struggle in (c) Fundamental Duties
the form of the Home Rule League. (d) Preamble
3. The myth of white supremacy was
busted by 1st World War.
35. Consider the following statements regarding
Which of the statements given above is/are
correct? the Hand-guided Cloning Technique:
(a) 1 and 3 only 1. In this technique, oocytes are matured
(b) 2 and 3 only in-vitro and enucleated using a hand
(c) 1 and 2 only held fine blade.
(d) 3 only
2. Recently, the National Dairy Research
Institute has developed a buffalo calf
32. Consider the following statements with
reference to mica in India: named ‘Tejas’ using this technology.
1. Mica is categorised as a minor mineral. Which of the statements given above is/are
2. Rajasthan has the highest reserves of correct?
mica in India. (a) 1 only
3. Most of the mica deposits in India
(b) 2 only
produce the muscovite variety or potash
(c) Both 1 and 2
mica.
Which of the statements given above is/are (d) Neither 1 nor 2
correct?
(a) 2 only 36. With reference to ‘fishing cat,’ a mammal
(b) 3 only found in India, which of the following
(c) 1 and 3 only
statements is/are correct?
(d) 2 and 3 only
1. It is the only wild cat species in India
33. Which of the following statements is/are that is a wetland specialist.
correct with reference to Mahatma Gandhi 2. It is categorised as Vulnerable on the
and Rabindranath Tagore? IUCN red list.
1. Tagore supported the Non-Cooperation 3. In India it is found only on the
movement call by Mahatma Gandhi as it
Himalayan foothills along the Ganga and
can bring Swaraj in the future.
2. Tagore criticized undue emphasis given Brahmaputra river valleys.
to Charkha by Mahatma Gandhi. Select the correct answer using the code
Select the correct answer using the code given below.
given below. (a) 1 only
(a) 1 only (b) 2 only
(b) 2 only
(c) 2 and 3 only
(c) Both 1 and 2
(d) Neither 1 nor 2 (d) 1 and 2 only

8 www.visionias.in ©Vision IAS

DEEPAK PHOTOSTAT 9310521834


37. Which of the following statements correctly 40. Consider the following statements with
describes The Tuting-Tidding Suture Zone reference to the 'Green Tax':
(TTSZ)? 1. Green tax would be levied only on
(a) It is a major part of Eastern Himalaya, transport vehicles to curb pollution in
where the Himalaya takes a sharp the country.
southward bend and connects with Indo
2. Electric vehicles and those running on
Burma range.
alternate fuels like CNG and ethanol
(b) It is a major part of Himalaya in the
would be exempted.
Ladakh region where Indian and Asian
Which of the statements given above is/are
Plates are joined.
(c) It is the subsidence of the western flank correct?
of the Indian Peninsula leading to its (a) 1 only
submergence below the sea. (b) 2 only
(d) It is a narrow belt ranging between 8-10 (c) Both 1 and 2
km parallel to the Shiwalik foothills at (d) Neither 1 nor 2
the break-up of the slope.
41. Photonics is the science and technology of
38. With reference to the Bar Council of India, generating, controlling, and detecting
consider the following statements: particles of light. Which of the following are
1. It is a statutory body created by
the possible applications of photonics?
Parliament to regulate and represent the
1. Diagnosis of diseases
Indian bar.
2. Fast data communication
2. It prescribes the standards of
3. Improve energy efficiency of lighting
professional conduct and etiquette of
advocates in Indian courts. 4. Detection of structural defects in
Which of the statements given above is/are buildings
correct? Select the correct answer using the code
(a) 1 only given below.
(b) 2 only (a) 3 and 4 only
(c) Both 1 and 2 (b) 1 and 2 only
(d) Neither 1 nor 2 (c) 1, 3 and 4 only
(d) 1, 2, 3 and 4
39. In the context of 'vermicompost', which of
the following is/are its advantages?
42. With reference to 'Smart Contracts', consider
1. It helps in raising the pH level of the soil
the following statements:
2. It helps in the reduction of the Carbon-
1. It is an agreement between two people in
Nitrogen ratio
3. Vermicompost can be suitably practised the form of computer code.
under a wide range of temperatures. 2. It can be easily edited and modified.
Select the correct answer using the code Which of the statements given above is/are
given below. correct?
(a) 1 only (a) 1 only
(b) 1 and 2 only (b) 2 only
(c) 2 and 3 only (c) Both 1 and 2
(d) 1, 2 and 3 (d) Neither 1 nor 2
9 www.visionias.in ©Vision IAS

DEEPAK PHOTOSTAT 9310521834


43. With reference to ‘Torrefaction technology’ 45. Recently, Indian scientists have brought out
consider the following statements: a biopesticide to control the Panama disease.
In this context, which of the following
1. It is a process that can convert rice statements is/are correct regarding Panama
stubble into bio-coal. Disease?
2. This biomass is heated to more than 1. It is a fungal disease that afflicts banana
plants.
800°C in a low oxygen environment. 2. It infects the plants through its root and
3. Combustible gases released during the stops water and essential nutrients from
being transported.
process can be used to generate the
Select the correct answer using the code
required heat. given below.
Which of the statements given above are (a) 1 only
(b) 2 only
correct?
(c) Both 1 and 2
(a) 1 and 2 only (d) Neither 1 nor 2
(b) 2 and 3 only
46. Which of the following are the government
(c) 1 and 3 only
initiatives for the promotion of the Chemical
(d) 1, 2 and 3 and Petrochemical Sector in India?
1. Increase in import duty on feedstock
such as ‘Naphtha'.
44. Which of the following statements is/are
2. 100% FDI is allowed under the
correct with reference to the Martial law automatic route in the chemicals sector.
under the Article 34 of Indian Constitution? 3. Mandating certificates for imported
chemicals to prevent the dumping of
1. It empowers the Parliament to indemnify
cheap chemicals.
any person for any act done by him in Select the correct answer using the code
connection with the maintenance of given below.
(a) 1 and 2 only
order in any area where martial law was
(b) 2 and 3 only
in force. (c) 1 and 3 only
2. All constitutional writs are suspended in (d) 1, 2 and 3

the area where martial law is declared.


47. Which of the following is/are the
3. Martial Law has never been imposed in components of External Debt of India?
post-independent India. 1. External Commercial Borrowings
2. FII investment in Government Securities
Select the correct answer using the code
3. Borrowing through Masala Bonds
given below. Select the correct answer using the code
(a) 1 and 3 only given below.
(a) 1 and 2 only
(b) 3 only
(b) 2 only
(c) 1, 2 and 3 (c) 1 and 3 only
(d) 2 and 3 only (d) 1, 2 and 3

10 www.visionias.in ©Vision IAS

DEEPAK PHOTOSTAT 9310521834


48. Consider the following statements regarding 51. Ranbir Singh Committee was recently
'Committee on private member's bill': constituted in the context of
1. It is a special committee of the Lok
(a) regulation of Over The Top (OTT)
Sabha.
media services.
2. In Lok Sabha, committee is presided
over by deputy speaker of house. (b) the revival of the economy after COVID
Which of the statements given above is/are slowdown.
correct? (c) regulation of cryptocurrency in India.
(a) 1 only
(d) reforms in criminal law.
(b) 2 only
(c) Both 1 and 2
(d) Neither 1 nor 2 52. Consider the following statements with
reference to the Bureau of Indian Standards
49. It is an Indian Chinese community living in (BIS):
the Northeast region, mostly in the states of
1. The Bureau consists of members from
Assam, Tripura, Mizoram. This community
is recognized as a Particularly Vulnerable both the Central and State government.
Tribal Group in Tripura. They perform the 2. It works under the aegis of the Ministry
well-known Hojagiri folk dance. In of Commerce and Industry.
Mizoram, they have been targeted by ethnic
Which of the statements given above is/are
organizations demanding its exclusion from
correct?
electoral rolls, forcing the tribe to flee to
Tripura. (a) 1 only
Which of the following tribal group is being (b) 2 only
described in the above passage? (c) Both 1 and 2
(a) Angami Tribe
(d) Neither 1 nor 2
(b) Lepcha Tribe
(c) Bru Tribe
(d) Mizo Tribe 53. Which of the following provisions of the
constitution of India help the courts in
50. Consider the following statements: examining and determining the
1. Under Article 131, Supreme Court has
constitutional validity of a law?
the original and exclusive jurisdiction to
deal with federal disputes in India. 1. Fundamental Rights
2. All disputes involving the 2. Directive Principle of State Policy
constitutionality of a central law are 3. Fundamental Duties
dealt with under Article 131.
4. Preamble
3. A central law cannot be challenged in
Select the correct answer using the code
the State High Courts.
Which of the statements given above is/are given below.
correct? (a) 1 only
(a) 1 only (b) 1 and 4 only
(b) 2 and 3 only
(c) 2 and 3 only
(c) 1 and 3 only
(d) None (d) 1, 2, 3 and 4

11 www.visionias.in ©Vision IAS

DEEPAK PHOTOSTAT 9310521834


54. Consider the following statements regarding 56. Consider the following statements with

Anushilan Samiti: respect to the Sanyasi rebellion:


1. It took place in Bengal after the great
1. The Dacca Anushilan and the Jugantar
famine of 1770.
were offshoot secret organizations from 2. Participants were originally tribals
a single parent body founded by whose forest-based livelihoods were
negatively impacted by the Forest laws
Promotha Mitter.
of the British.
2. Dacca Anushilan Samiti was responsible
3. The immediate cause of the rebellion
for the Barrah Dacoity. was the restrictions imposed by the
Which of the statements given above is/are British upon pilgrims visiting holy
places.
correct?
4. Anandamath, a novel by Bankim
(a) 1 only
Chandra Chattopadhyay, is based on the
(b) 2 only Sanyasi Revolt.

(c) Both 1 and 2 Which of the statements given above are


correct?
(d) Neither 1 nor 2
(a) 1, 2 and 3 only
(b) 2 and 4 only
55. Which of the following statements about (c) 1, 3 and 4 only

Permafrost are correct? (d) 1, 2, 3 and 4

1. There is approximately twice as much


57. How is the Later Vedic Period different from
carbon in permafrost than is currently in the Rigvedic period?
the Earth’s atmosphere. 1. The Rigvedic Aryans were pastoralists
wherein Later Vedic Period agriculture
2. Thawing of permafrost may lead to an
became the mainstay of the economy.
outbreak of bacterial and viral diseases.
2. Social differentiation in the form of the
3. Thawing of permafrost can damage Varna system was prevalent in both the
houses, roads, gas pipelines, and other Early and Later Vedic periods.
3. The early Rigvedic gods like Indra and
infrastructure.
Agni lost their importance during the
Select the correct answer using the code later Vedic period.
given below. Which of the statements given above are

(a) 1 and 3 only correct?


(a) 1 and 2 only
(b) 1 and 2 only
(b) 2 and 3 only
(c) 2 and 3 only (c) 1 and 3 only
(d) 1, 2 and 3 (d) 1, 2 and 3

12 www.visionias.in ©Vision IAS

DEEPAK PHOTOSTAT 9310521834


58. With reference to the Construction & 60. Various state governments have proposed
Demolition (C&D) Waste Management private sector reservations in their respective
Rules, 2016, consider the following states. In this context, consider the following
statements: statements:
1. These rules apply to the waste generated 1. Bill recommending such reservation
from the construction of new buildings must be reserved for the President's
but not to the repair of any existing approval.
buildings. 2. Supreme Court has capped the
2. Defence establishments are exempt from reservation to 50 percent in both the
these rules for generating any public and private sectors.
construction and demolition waste Which of the statements given above is/are
arising from the construction of any civil correct?
structure. (a) 1 only
3. As per these rules Central Pollution (b) 2 only
Control Board (CPCB) shall prepare (c) Both 1 and 2
operational guidelines related to (d) Neither 1 nor 2
environmental management of
construction and demolition waste 61. Migration is an important aspect of the lives
management. of many animals. In this context, consider
Which of the statements given above is/are the following pairs:
correct? Type of Fish Type of Migration
(a) 3 only 1. Anadromous : Born in freshwater but
(b) 1 and 3 only fish migrate into the
(c) 1 and 2 only saltwater where they
(d) 1, 2 and 3 grow into adults.
2. : Born in saltwater but
59. Consider the following statements regarding Potamodromous migrate into the
the Mesenchymal Stem Cells (MSCs) : fish freshwater where they
1. They are adult stem cells which can be grow into adults
isolated from both humans and animals. 3. Catadromous : Born in freshwater and
2. They can be used to make specialized fish migration is also
cells found in the skeletal tissues. within freshwater.
Which of the statements given above is/are Which of the pairs given above is/are
correct? correctly matched?
(a) 1 only (a) 1 only
(b) 2 only (b) 1 and 2 only
(c) Both 1 and 2 (c) 2 and 3 only
(d) Neither 1 nor 2 (d) 1, 2 and 3

13 www.visionias.in ©Vision IAS

DEEPAK PHOTOSTAT 9310521834


62. South Asia Cooperative Environment 65. Consider the following statements regarding
Programme (SACEP) and South Asia the sittings of the houses of the Parliament of
Wildlife Enforcement Network (SAWEN) India:
has been set up for better cooperation in the
1. A sitting of the Lok Sabha is duly
field of environment and to combat wildlife
constituted only when it is presided over
crime in South Asia respectively. Which of
by the Speaker.
the following countries are members of both
2. The presence of at least one Minister is
these bodies?
1. The Maldives necessary for constituting a sitting of the

2. Pakistan House
3. India 3. The assembly of the two Houses of
4. Afghanistan Parliament to hear the President’s
Select the correct answer using the code Address constitutes a sitting of the Rajya
given below. Sabha.
(a) 1 and 2 only
Which of the statements given above is/are
(b) 1, 3 and 4 only
correct?
(c) 2, 3 and 4 only
(a) 1 only
(d) 1, 2, 3 and 4
(b) 1 and 2 only

63. With reference to the period of the Gupta (c) 2 and 3

kingdom in ancient India, the terms Bhaga, (d) None


Hiranya and Udianga refer to
(a) religious rituals 66. Consider the following statements in respect
(b) measurements of land of the International Climate Ambition
(c) different kinds of taxes Summit:
(d) coins of different money value
1. It was organized and co-hosted by the
World Economic Forum and World
64. Consider the following constitutional posts:
Wildlife Fund.
1. Election Commission of India
2. Comptroller and Auditor General of 2. It aimed to highlight the achievements of

India public as well as private agencies

3. Attorney General of India towards carbon reduction and enhancing


4. Judges of Supreme Court the climate finance.
Which of the above are mentioned in both Which of the statements given above is/are
the Second and Third Schedule of the correct?
Constitution? (a) 1 only
(a) 1 and 2 only
(b) 2 only
(b) 2 and 3 only
(c) Both 1 and 2
(c) 3 and 4 only
(d) Neither 1 nor 2
(d) 2 and 4 only
14 www.visionias.in ©Vision IAS

DEEPAK PHOTOSTAT 9310521834


67. Trade deficit between India and China 70. “It is a process of inserting of a part of one
declined to a five year low in 2020. Which plant in to another in such a way that a
of the following are the reasons for the union will take place and combination will
decline in trade deficit? continue to grow as a single plant. Plum is
1. Decline in China's exports to India. grown through such a method of plant
2. Increase in India's exports to China. propagation.”
3. Sharp increase in exports of Indian iron Which of the following methods of plant
propagation is being described in the above
and steel to China.
passage?
Select the correct answer using the code
(a) Cutting
given below.
(b) Layering
(a) 1 only
(c) Budding
(b) 2 and 3 only
(d) Grafting
(c) 1 and 3 only
(d) 1, 2 and 3 71. With reference to International Investment
Position (IIP), consider the following
68. Consider the following statements with statements:
reference to the Competition Commission of 1. It shows an economy’s stock of external
India: financial assets and liabilities at a
1. It is a statutory body established by the particular point of time
central government to ensure freedom of 2. A positive IIP means that the nation is
trade in the markets of India. the debtor and a negative IIP means that
2. It has the power to punish those it is a creditor.
indulging in anti-competitive practices Which of the statements given above is/are
3. It can take up suo moto investigation for correct?
alleged price cartelisation of products. (a) 1 only
Which of the statements given above is/are (b) 2 only
(c) Both 1 and 2
correct?
(d) Neither 1 nor 2
(a) 1 only
(b) 2 and 3 only
72. 'This performance is a derivative of
(c) 1 and 3 only
Koodiyattam, the ancient Sanskrit theatre.
(d) 1, 2 and 3
The text for this art form is
Sreekrishnacharitham that comprises 217
69. Which of the following were built during the slokas taken from various scriptures. It starts
reign of Pallavas? with a description of Mathura and
1. Kailasanath temple at Kanchipuram culminates with the abduction of Subhadra.
2. Virupaksha temple at Pattadakal It is performed by a single female actor
3. Shore temple at Mamallapuram assuming the roles of different characters of
4. Mandagapattu rock cut temple the story.'
Select the correct answer using the code Which art form is being described in the
given below. passage given above?
(a) 1 and 3 only (a) Swang
(b) 2 and 4 only (b) Therukoothu
(c) 1, 3 and 4 only (c) Yakshgana
(d) 2, 3 and 4 only (d) Nangiarkoothu

15 www.visionias.in ©Vision IAS

DEEPAK PHOTOSTAT 9310521834


73. Consider the following statements with 76. Multiple Invasive Alien Species (IAS) have
reference to peatlands: become a threat to ecosystems worldwide. In
1. They are the largest natural terrestrial this context, which of the following are the
carbon store. common characteristics of IAS?
2. Peatland accounts for less than ten per 1. Rapid reproduction and growth
cent of the global land surface.
2. High dispersal ability
3. They can be classified as wetlands under
3. Phenotypic plasticity
the Ramsar Convention.
Select the correct answer using the code
4. Peatland occurs in every climatic zone.
given below.
Which of the statements given above is/are
correct? (a) 1 and 2 only

(a) 1 only (b) 1 and 3 only


(b) 1, 3 and 4 only (c) 2 and 3 only
(c) 2, 3 and 4 only (d) 1, 2 and 3
(d) 1, 2, 3 and 4
77. Recently 100th death Anniversary of Bal
74. Which of the following statements explain Gangadhar Tilak was observed. Which of
the term ‘Equality before law’ under the the following statements is not correct
Article 14 of Indian constitution? regarding Bal Gangadhar Tilak?
1. Absence of any special privileges in
(a) He was a contemporary of Swami
favour of any person.
Vivekananda.
2. No person is above the law.
(b) His concept of Swarajya was based on
3. Like should be treated alike without
the Vedanta philosophy.
discrimination
(c) He vehemently criticized British rule
Select the correct answer using the code
given below. through his Marathi publication “The
(a) 1 and 2 only Mahratta”.
(b) 1 only (d) He published 'The Orion' to establish the
(c) 1, 2 and 3 primacy of the Vedic literature.
(d) 2 and 3 only
78. Recently govt has proposed to set up India’s
75. Consider the following pairs: 4th ‘Bird Ringing Station’ in Bihar. Which
Type of soil Associated region one of the following is not the purpose to
1. Alluvial soil : Piedmont plains establish a bird ringing station?
2. Laterite soil : High temperature and
(a) To observe, monitor and research on
heavy rainfall
migratory birds.
3. Red soil : Low rainfall
(b) To study population changes and species
Which of the pairs given above is/are
decline.
correctly matched?
(a) 1, 2 and 3 (c) In situ conservation of critically

(b) 1 only endangered avian species.


(c) 2 and 3 only (d) To address the problem of bird
(d) 3 only conservation.
16 www.visionias.in ©Vision IAS

DEEPAK PHOTOSTAT 9310521834


79. Consider the following statements with 82. "It was written by a Buddhist poet. It
reference to the Producer Price Index (PPI): describes a story of a dancer who preserves
1. It measures the change in average retail her chastity from a Prince and becomes a
prices that a consumer pays for the Buddhist nun to preach its doctrines. who
goods. became a Buddhist nun. It gives glimpses of
2. PPI excludes indirect taxes. the development of the fine arts in the
Which of the statements given above is/are Sangam age."
correct? Which of the following works is being
(a) 1 only described in the above-given passage?
(b) 2 only (a) Silappadikaram
(c) Both 1 and 2 (b) Manimekalai
(d) Neither 1 nor 2 (c) Tolkappiyam
(d) Ettutogai
80. Which of the following is the objective of
the recently constituted DISHA fund? 83. Which of the following indicates high
(a) To promote indigenous manufacturing stability of banks?
of unmanned aerial vehicles (UAVs). 1. High Capital Adequacy Ratio
(b) To promote internship in Intellectual 2. Low Non-performing loans to total loans
Property Rights (IPRs) for Self- 3. High Loan to Deposit ratio
Employment. Select the correct answer using the code
(c) To encourage startups with a focus on given below.
the marketing of tribal goods and (a) 1 and 2 only
handicrafts. (b) 2 and 3 only
(d) To provide subsidy for individual (c) 1 and 3 only
farmers complying with stubble burning (d) 1, 2 and 3
guidelines.
84. Consider the following statements regarding
81. Which of the following are famous for Jain 'GAVI, the Vaccine Alliance':
caves? 1. It is a public-private global health
1. Udaygiri partnership with the goal of increasing
2. Badami access to immunization in poor countries
3. Ellora 2. It was formed as an emergency response
4. Ravan phadi system to fight the COVID pandemic.
Select the correct answer using the code Which of the statements given above is/are
given below. correct?
(a) 1 and 3 only (a) 1 only
(b) 2 and 4 only (b) 2 only
(c) 1, 2 and 3 only (c) Both 1 and 2
(d) 1, 2, 3 and 4 (d) Neither 1 nor 2
17 www.visionias.in ©Vision IAS

DEEPAK PHOTOSTAT 9310521834


85. Which of the following statements is/are 87. Consider the following statements with
correct regarding the States and Union respect to imposition of President's rule:
Territories (UTs) in India? 1. The proclamation imposing President’s
1. All State legislative assemblies and all Rule is subject to judicial review.
Union Territories with legislature have 2. It can only be imposed on states and not
nominated members.
Union Territories.
2. All states and only the Union Territories
3. It can be proclaimed on the advice of the
of Delhi and Puducherry have
Council of Ministers tendered under
representation in Rajya Sabha.
Article 74.
Select the correct answer using the code
Which of the statements given above are
given below.
correct?
(a) 1 only
(a) 1 and 2 only
(b) 2 only
(b) 1 and 3 only
(c) Both 1 and 2
(d) Neither 1 nor 2 (c) 1, 2 and 3

(d) 2 and 3 only

86. With reference to the Indian economy,


consider the following statements: 88. Consider the following statements regarding

1. Angel fund is a money pool created by the National Plan for Conservation of
high net worth individuals for investing Aquatic Eco-systems (NPCA) scheme:
in business start-ups. 1. Ministry of Jal Shakti is the
2. Fund of Funds (FOF) is an investment implementing agency of this scheme.
strategy of holding a portfolio of other 2. It deals with the conservation and
investment funds rather than investing
management of lakes as well as
directly in stocks.
wetlands.
3. Debt fund is an Alternative Investment
3. Fisheries development is one of the
Fund (AIF) which invests primarily in
activities covered under this scheme.
debt securities of listed companies.
Which of the statements given above is/are
Which of the statements given above is/are
correct?
correct?
(a) 2 only
(a) 1 only
(b) 2 and 3 only
(b) 2 and 3 only
(c) 1 and 3 only (c) 1 and 3 only

(d) 1, 2 and 3 (d) 1, 2 and 3

18 www.visionias.in ©Vision IAS

DEEPAK PHOTOSTAT 9310521834


89. Consider the following statements regarding 92. With reference to Quasars, consider the
the differences between cloud computing following statements:
and grid computing: 1. It is an astronomical object found at the
1. While cloud computing does not require centres of galaxies in which a
setting up of software, in grid computing supermassive black hole is surrounded
installation of the software component is by a gaseous accretion disk.
essential. 2. It functions like a black hole and thus
2. In comparison to the grid computing can only be detected by watching for its
cloud computing maintains the resources effects on nearby stars and gas.
centrally. Which of the statements given above is/are
Which of the statements given above is/are correct?
correct?
(a) 1 only
(a) 1 only
(b) 2 only
(b) 2 only
(c) Both 1 and 2
(c) Both 1 and 2
(d) Neither 1 nor 2
(d) Neither 1 nor 2

93. Which of the following statement is not


90. With reference to ‘Laurentian type of
correct with respect to the Great Rann of
climate’, which of the following is not
Kutch Biosphere Reserve?
correct?
(a) It is famous for Asiatic Lion.
(a) It has features of both the maritime and
(b) Tropic of Cancer Passes through it.
the continental climates.
(c) This biosphere reserve is drained by
(b) It is some times referred to as the Gulf
Luni River.
type of climate.
(c) It has cold, dry winters and warm, wet (d) The Banni grasslands are found in this

summers. biosphere reserve.

(d) In the Southern Hemisphere, this


climatic type is absent. 94. Consider the following statements with
reference to the afforestation of sand dunes:
91. Which of the following factors affect seed 1. The plant or the tree species to be used
germination? for afforestation must be drought
1. Permeability to water resistant.
2. Availability of oxygen 2. The tree species should be able to
3. Size of the seed withstand high temperatures and severe
4. Burial depth of the seed frost.
Select the correct answer using the code Which of the statements given above is/are
given below. correct?
(a) 1, 2 and 3 only (a) 1 only
(b) 2 and 4 only (b) 2 only
(c) 1, 3 and 4 only (c) Both 1 and 2
(d) 1, 2, 3 and 4 (d) Neither 1 nor 2
19 www.visionias.in ©Vision IAS

DEEPAK PHOTOSTAT 9310521834


UPSC CSE Free Material (OPTIMISTIC IAS)
95. With reference to the Forest Survey of India 98. Consider the following statements with
(FSI), consider the following statements: respect to solifluction:
1. It releases the State of Forest Report 1. It is a slow downslope flowing soil mass
or fine-grained rock debris saturated or
biennially, providing an assessment of
lubricated with water.
the latest forest cover in the country. 2. It is common only in the tropical
2. It carries out an inventory of trees climatic regions.
outside forest areas. Which of the statements given above is/are
3. Its headquarters is located at Dehradun. correct?
Which of the statements given above are (a) 1 only
(b) 2 only
correct?
(c) Both 1 and 2
(a) 1 and 3 only (d) Neither 1 nor 2
(b) 2 and 3 only
(c) 1 and 2 only 99. With reference to 'Privy Purse' in India,
(d) 1, 2 and 3 consider the following statements:
1. It was a monetary benefit provided to
erstwhile princely states in return of
96. It is considered the world’s first electronic
their sovereignty claim.
musical instrument. It has a vertical antenna 2. It was abolished by the 26th
for controlling the tone and a looped antenna Constitutional Amendment Act, 1971.
for controlling the volume. It generates Which of the statements given above is/are
sound by modulation of electromagnetic correct?
(a) 1 only
waves and this is done without the player
(b) 2 only
having to even touch the instrument. (c) Both 1 and 2
Which of the following musical instrument (d) Neither 1 nor 2
is being described in the above passage?
(a) AlphaSphere 100. With reference to the practise of
(b) Kaosscilator 'Untouchability', consider the following
statements
(c) Theremin
1. Untouchability is not defined in the
(d) The Reactable Constitution of India.
2. Constitution provides practise of
97. Which of the following is a common feature untouchability as one of the grounds for
between the federation system in India and disqualification of a member of
Parliament.
the USA?
3. It is a non-bailable and cognizable
(a) Equality of representation of states in
offence under the Protection of Civil
upper house of the federal legislature. Rights Act, 1955.
(b) Vesting of residuary powers in the Which of the statements given above are
Union. correct?
(c) Union cannot be destroyed by any state (a) 1 and 3 only
(b) 2 and 3 only
seceding from the Union at its will.
(c) 1 and 2 only
(d) Consent of state is required for altering (d) 1, 2 and 3
its boundaries by Parliament.

Copyright © by Vision IAS


All rights are reserved. No part of this document may be reproduced, stored in a retrieval system or transmitted
in any form or by any means, electronic, mechanical, photocopying, recording or otherwise, without prior
20 www.visionias.in ©Vision IAS
permission of Vision IAS.

DEEPAK PHOTOSTAT 9310521834


UPSC CSE Free Material (OPTIMISTIC IAS)

VISIONIAS
www.visionias.in

ANSWERS & EXPLANATIONS


GENERAL STUDIES (P) TEST – 3206 (2021)

Q 1.D
• Edwin Montagu was appointed the Secretary of State for India in 1917 and remained in that office
till 1922. He was critical of the manner in which India was governed.
• On 20th August 1917, Montagu presented the historic Montagu Declaration (August Declaration) in
the British Parliament.
o This declaration advocated for the increasing the association of Indians in every branch of
administration.
o He also laid the foundation for 'gradual development' of self-governing institutions with a view to the
progressive realisation of responsible government in India as an integral part of the British empire in
long run. Hence statement 1 is not correct.
• In 1917, Montagu visited India and held talks with the various representatives of Indian polity including
Mahatma Gandhi and Muhammad Ali Jinnah. He, along with the Governor-General of India Lord
Chelmsford, brought out a detailed report titled Constitutional Reforms in India, also called the
Montagu-Chelmsford Report. This report was published on 8th July 1918. This report became the
basis for the Government of India Act 1919 (alternatively called the Montagu-Chelmsford Reforms
or Montford Reforms). The report was rejected by most Indian leaders. Annie Besant (Born on October
1st, 1847) referred to it as ‘unworthy to be offered by England or to be accepted by India’.
• The GOI Act, 1919 though introduced elections but the franchise was partial in nature, not universal. Only
certain people who were propertied or had titles, etc. or held office could vote. Hence statement 2 is not
correct.
Q 2.B
• Genetic information flows from DNA into protein, the substance that gives an organism its form. This
flow of information occurs through the sequential processes of transcription (DNA to RNA) and
translation (RNA to protein). Transcription occurs when there is a need for a particular gene product at a
specific time or in a specific tissue. Hence statement 1 is not correct and statement 2 is correct.
• Translation is the process of translating the sequence of a messenger RNA (mRNA) molecule to a
sequence of amino acids during protein synthesis. The genetic code describes the relationship between
the sequence of base pairs in a gene and the corresponding amino acid sequence that it encodes.
• There are several different types of RNA, each having different functions in the cell. The structure of
RNA is similar to DNA with a few small exceptions. For one thing, unlike DNA, most types of RNA,
including mRNA, are single-stranded and contain no complementary strand. Second, the ribose sugar in
RNA contains an additional oxygen atom compared with DNA. Finally, instead of the base thymine, RNA
contains the base uracil. This means that adenine will always pair up with uracil during the protein
synthesis process.

1 www.visionias.in

DEEPAK PHOTOSTAT 9310521834


Q 3.C
• In 1826 a Jury Bill for India was passed, which came into operation in the beginning of 1827. The bill
introduced religious distinctions into the judicial system of this country.
• It provided that any Natives either Hindu or Mahommedan can be tried by Christians, either European or
Native, while Christians including Native converts, are exempted from the degradation of being tried
either by a Hindu or Mussalman juror, however high he may stand in the estimation of society. This
Bill also denies both to Hindus and Mussalmans the honour of a seat in the Grand Jury, even in the
trial of fellow Hindus or Mussalmans. Hence statement 1 is correct.
• A champion of civil liberties and freedom, Raja Rammohan Roy prepared and sent up to both Houses of
Parliament petitions against it signed by Hindus and Mahommedans. On this occasion Rammohan Roy
took his stand against the injustice and injudiciousness of making religious distinctions. Hence statement
2 is correct.

Q 4.C
• Bioceramics: They are ceramic materials specially developed for use as medical and dental
implants. They are usually used to replace hard tissue in the body like bone and teeth. Common
bioceramics are alumina, zirconia and a form of calcium phosphate known as hydroxyapatite. Hence,
statement 1 is not correct.
• Bioactive and bio-inert: Bioactive ceramics interact with the body so that tissue bonding and eventual
incorporation into the body occurs after a time. Calcium phosphate-based bioceramics are bioactive.
Bioinert ceramics do not interact with the body’s environment apart from causing an initial ‘fibrous
tissue’ reaction, which coats the ceramic. Alumina and zirconia-based ceramics are classed as bio-
inert. Hence statement 2 is not correct and statement 3 is correct.
• Raw materials: Alumina (Al2O3) is a white powder. When shaped, compressed and heated to a high
temperature (sintering), the ceramic that results has high density, high strength, excellent corrosion
resistance, good biocompatibility and high wear resistance. In addition, it can be machined, ground and
polished to a high-quality product. Alumina bioceramics are used as replacement parts in hip and knee
operations. The inertness of the ceramic, its high wear resistance and its excellent biocompatibility make
it the ceramic of choice. The high load-bearing properties of alumina also makes it an ideal ceramic for
dental implants.
• Zirconia (ZrO2) is also a white powder. Like alumina, it can be compressed and sintered into a very
strong ceramic. Unlike alumina, its wear-resistance properties are not as good. By adding yttrium oxide
and small amounts of magnesium oxide, a better-wearing bioceramic called Y-TZP can be made. Zirconia
bioceramics do not have the high wear resistance of alumina bioceramics and are not as widely used in hip
joint applications. However, their fracture toughness and bending strength give them additional qualities.
• Calcium phosphate or hydroxyapatite – Ca10(PO46(OH)2 – is the principal component of natural
bone in the body. Ceramics made from synthetic calcium phosphate can also be used in medical
applications. The problem is that these ceramics are not as strong as alumina or zirconia ceramics.
Calcium phosphate ceramics can bond to bone and promote bone growth at their surfaces. A popular use
of these ceramics is as coatings on dental and orthopaedic implants. For example, titanium tooth root pegs
coated with hydroxyapatite (a form of calcium phosphate) give a longer lasting implant than pegs that
have been glued or cemented in place. The hydroxyapatite binds chemically with living bone because it is
a bioactive ceramic.
• Bovine hydroxyapatite: Often as a result of injuries sustained in road crash accidents, bone
reconstruction operations are needed. A novel method involving the use of bovine hydroxyapatite has
been recently successfully tested in sheep and dogs.

Q 5.A
• There are several different types of vaccines. Each type is designed to teach your immune system how to
fight off certain kinds of germs—and the serious diseases they cause. When scientists create vaccines,
they consider:
o How your immune system responds to the germ
o Who needs to be vaccinated against the germ
o The best technology or approach to creating the vaccine
o Based on a number of these factors, scientists decide which type of vaccine they will make.
• There are several types of vaccines, including:
• Inactivated vaccines: Inactivated vaccines use the killed version of the germ that causes a disease.

2 www.visionias.in ©Vision IAS

DEEPAK PHOTOSTAT 9310521834


o Inactivated vaccines usually do not provide immunity (protection) that is as strong as live
vaccines. So you may need several doses over time (booster shots) in order to get ongoing immunity
against diseases. Hence, statement 2 is not correct.
o Inactivated vaccines are used to protect against Hepatitis A, Flu (shot only), Polio (shot only), Rabies
• Live-attenuated vaccines: Live vaccines use a weakened (or attenuated) form of the germ that
causes a disease. Hence, statement 1 is correct.
o Because these vaccines are so similar to the natural infection that they help prevent, they create a
strong and long-lasting immune response. Just one or two doses of most live vaccines can give you
a lifetime of protection against a germ and the disease it causes.
o But live vaccines also have some limitations. For example they need to be kept cool, so they don’t
travel well.
o Live vaccines are used to protect against Measles, mumps, rubella (MMR combined vaccine),
Rotavirus, Smallpox, Chickenpox, Yellow fever.

Q 6.C
• The Goddess Mata Ragnya Devi is symbolized as a sacred spring at Tula Mula village, near (27 km)
Srinagar, Kashmir in J&K India. The spring of Ksheer Bhawani is quite large and has always been
held in veneration by the Brahman population of Srinagar. The devotees of the Goddess Mata
Ragnya fast and gather here on the eighth day of the full moon in the month of May/ June every year
when, according to belief, the Goddess changes the colour of the spring's waters, which are ascribed to
different manifestations of the central deity. Turning the colour into shades of black is supposed to signal
approaching disaster. The term “kheer” refers to rice pudding offered in the spring to propitiate the
goddess, which became part of the name of the temple. Swami Vivekananda visited the Tula Mula village
and stayed for a week in 1898 and was astonished by the color change which he cited as a miracle. Hence
option (c) is the correct answer.
• Ambubachi Mela is one of the biggest congregations of eastern India. It is the most important festival of
the Kamakhya temple and is celebrated in the month of June every year. It is more of a ritual of
austerities, a festival celebrated with Shakti rites. The belief is that Kamakhya embodies the mother cult,
the Shakti. During the period of Ambubachi from the seventh to the tenth day of the Hindu month of
"Asadha", the doors of the shrine are closed to all as it is believed that Goddess Kamakhya goes through
the annual cycle of menstruation. On the twelfth day, the doors are opened ceremonially and a big fair
held at the temple premises on that day.
• Jude Sheetal is the name given to the Maithili New year celebrated in the month of April This is also
called the Nirvana Mesh Sankranti or the Tiruhata New Year.
• Herath is considered as one of the most important festivals of Kashmiri Pandits in the month of
February/March. The devotees pray to lord Shiva and visit temples of Shankracharya on the hills of Takht
–e Suleman. An interesting fact about Herath is that it is celebrated one day before the Maha-Shivratri
celebrations in India.

Q 7.A
• Sambhar Salt Lake is India's largest inland salt lake, located 80 km southwest of the city of Jaipur. It
surrounds the historical Sambhar Lake Town. The lake receives water from the rivers Medtha, Samaod,
Mantha, Rupangarh, Khari, and Khandela.
• The lake is an extensive saline wetland, with water depth fluctuating from as few as 60 centimetres during
the dry season to about 3 meters at the end of the monsoon season.
• The lake is elliptically shaped with a length of approximately 35.5 km and a breadth varying between
3 km and 11 km. It is surrounded on all sides by the Aravali hills. Hence, statement 1 is correct.
• Sambhar lake has been designated as a Ramsar site because the lake is a key wintering area for tens of
thousands of pink flamingos and other birds that migrate from northern Asia and Siberia. The specialized
algae and bacteria growing in the lake provide striking watercolours and support the lake ecology
which, in turn, sustains the migrating waterfowl. Algae and bacteria form a crucial component of
this lake's ecosystem. Hence, statement 2 is not correct.
• Gujarat is the largest producer of salt in India with a share of more than 70 per cent of India's total
salt production. Sambhar lake share is around 10 per cent of India's total salt production. Hence,
statement 3 is not correct.
• India is the third-largest producer of salt in the world. The top five salt-producing states in India are
Gujarat, Tamil Nadu, Rajasthan, Maharashtra, and Andhra Pradesh.

3 www.visionias.in ©Vision IAS

DEEPAK PHOTOSTAT 9310521834


Q 8.B
• Chaitanya (1486-1533) was the most prominent Vaishnava saint of Bengal. He popularized Krishna-
bhakti in many parts of Eastern India. His popularity as a religious personality was so great that he was
looked upon as an avatar (incarnation) of Krishna even in his life.
o Chaitanya’s concern, unlike that of Kabir, was not with bringing people to an understanding of a God,
beyond all creeds and formulations; it was to exalt the superiority of Krishna over all other deities.
It was, in other words, a revivalist, not a syncretic movement, a return to worship of Vishnu under one
of his most appealing forms, the loving ecstatic Krishna. Hence statement 2 is correct.
o Chaitanya disregarded all distinctions of caste, creed, and sex to give a popular base to Krishna
bhakti. His followers belonged to all castes and communities. One of his most favorite disciples was
Haridas who was a Muslim. He popularized the practice of sankirtan or group devotional singing
accompanied by ecstatic dancing.
o However, Chaitanya did not give up traditional Brahmanical values altogether. He did not
question the authority of the Brahmans and scriptures. He upheld the caste prejudices of his
Brahman disciples against the "lower" caste disciples. However, his disregard for caste distinctions in
the sphere of devotional singing promoted a sense of equality in Bengali life. Hence statement 3 is
not correct.
• Srimanta Sankardev (1449–1568) was a 15th–16th century Assamese saint-scholar, poet, playwright,
and figure of importance in the cultural and religious history of Assam, India.
o He is widely credited with building on past cultural relics and devising new forms of music (Borgeet),
theatrical performance (Ankia Naat, Bhaona), dance (Sattriya), literary language (Brajavali).
o He started the Bhagavatic religious movement also called Neo-Vaishnavite movement.
o The assembly of devotees he initiated evolved over time into monastic centers called Sattras, which
continue to be important socio-religious institutions in Assam and to a lesser extent in North Bengal.
• Both Chaitanya Mahaprabhu and Shankaradevaa were contemporaries. Hence statement 1 is
correct.

Q 9.A
• Recently in Anuradha Bhasin vs. Union of India wherein the validity of internet shutdown and
movement restrictions in J&K was challenged in the Hon'ble Supreme court, the Doctrine of
Proportionality was invoked. It stated that the appropriateness of a measure depends on its implication
upon the fundamental rights and the necessity of such measure. It is undeniable from the aforesaid holding
that only the least restrictive measure can be resorted to by the State, taking into consideration the facts
and circumstances. Lastly, since the order has serious implications on the fundamental rights of affected
parties, the same should be supported by sufficient material and should be amenable to judicial review.
• The Doctrine of Proportionality asserts that there must be a reasonable nexus between the desired
result and the measures taken to reach that goal. The action taken must not be shockingly
disproportionate to the consciousness of the court and the said action can then be challenged by way of
judicial review.
• For example - If in a workplace some workers remain absent from their duty then the punishment for it
must be proportional, that is, the employer may treat it as leave without pay and may warn them or may
even levy a fine but to dismiss them from service permanently would be disproportional. Hence option
(a) is the correct answer.
• Doctrine of proportionality finds its place in the Administrative law and is used at the stage of judicial
review.
• The principle of proportionality evaluates two aspects of a decision:
o Whether the relative merits of differing objectives or interests were appropriately weighed or "fairly
balanced"?
o Whether the measure in question was in the circumstances excessively restrictive or inflicted an
unnecessary burden on affected persons?
• The court while applying Doctrine of Proportionality is not concerned with the correctness of the
decision rather the method to reach such decision. The decision-making process involves attributing
relative importance to various aspects in the case and there the doctrine of proportionality enters.
• Proportionality' involves a "Balancing test" which keeps a check on the excessive or arbitrary
punishments or encroachment upon the rights and "Necessity test" which takes into account other less
restrictive alternates. Thus, it keeps a check on excessive or arbitrary decision by the government.
• The Doctrine of Proportionality checks whether different options involved in decision were considered
and the final measure taken is not excessively restrictive or inflict an unnecessary burden on affected
persons.
4 www.visionias.in ©Vision IAS

DEEPAK PHOTOSTAT 9310521834


Q 10.A
• There were eight schedules enshrined in Indian constitution during its commencement. The ninth schedule
became a part of the Constitution in 1951, when the document was amended for the first time.
• It was created by the First Constitutional (Amendment) Act, 1951 which added new Article 31B,
along with 31A in order to protect laws related to agrarian reform and for abolishing the
Zamindari system. Hence statements 1 and 2 are correct.
• Though the 1st Amendment (1951) to protect the laws included in it from judicial scrutiny on the
ground of violation of fundamental rights. However, in January 2007, the Supreme Court ruled that
there could not be any blanket immunity from judicial review of laws included in the Ninth Schedule.
• The court held that judicial review is a ‘basic feature’ of the constitution and it could not be taken away by
putting a law under the Ninth Schedule. It said that the laws placed under the Ninth Schedule after
April 24, 1973, are open to challenge in court if they violated fundamentals rights guaranteed under
Articles 14, 15, 19 and 21 or the ‘basic structure’ of the constitution.
• (As on April 24, 1973, that the Supreme Court first propounded the doctrine of ‘basic structure’ or ‘basic
features’ of the constitution in its landmark verdict in the Kesavananda Bharati Case). Hence statement 3
is not correct.
• Additional Information: The 76th Amendment Act of 1994 has placed the Tamil Nadu Reservations
Act of 1994 in the Ninth Schedule to protect it from judicial review as it provided for 69 per cent of
reservation, far exceeding the 50 per cent ceiling.

Q 11.B
• Mangroves are trees or shrubs that are found in the intertidal zone of coastlines or the area between the
coastal environment and the terrestrial environment. These florae are well-adapted to living in saline and
brackish environments, which is one of the reasons that they are so unique.
• There are about 70 species of mangroves found along tropical and subtropical coastlines, with a
particularly heavy presence in Asia, followed by Africa and South America.
• The largest area of mangroves is found in Indonesia, followed by Brazil, Australia, Nigeria, and Mexico.
• The Global Mangrove Alliance was co-founded by The Nature Conservancy (TNC), World Wildlife
Fund (WWF ), and Conservation International(CI). Hence, statement 1 is not correct
• The Global Mangrove Alliance is currently coordinated by members Conservation International, The
International Union for the Conservation of Nature, The Nature Conservancy, Wetlands International, and
World Wildlife Fund.
• The main target of the Alliance is to increase the global area of mangrove habitat by 20 percent
over the current area by the year 2030. Hence, statement 2 is correct.
• It also focuses on climate adaptation, climate mitigation, sustaining biodiversity, and improving human
well-being.

Q 12.C
• Exfoliation tors:
o Exfoliation is a result of weathering but not a process.
o Various minerals in rocks possess their own limits of expansion and contraction.
o With rising in temperature every mineral expands and pushes against its neighbour and as the
temperature falls, a corresponding contraction takes places because of diurnal changes in the
temperature, this internal movement among the mineral grains of the superficial layers of rocks takes
place regularly.
o This process is most effective in dry climates and high elevations where diurnal temperature changes
are drastic. However, these movements are very small, they make the rocks weak due to continued
fatigue.
o The surface layers of the rocks tend to expand more than the rocks at depth resulting in heaving and
fracturing parallel to the surface and their subsequent exfoliation from the surface results in smooth
rounded surfaces in rocks.
o In rocks like granites, smooth-surfaced and rounded small to big boulders called tors form due to such
exfoliation. Hence option (c) is the correct answer.
• Exfoliation domes:
o It is formed due to unloading and expansion.
o Removal of overlying rock load because of continued erosion causes vertical pressure release with the
result that the upper layers of the rock expand producing disintegration of rock masses.
o The fracture will develop roughly parallel to the ground surface. It can be measured hundred or
thousands of metres.
5 www.visionias.in ©Vision IAS

DEEPAK PHOTOSTAT 9310521834


Q 13.D
• The Framework for Strategic Partnership between the International Energy Agency (IEA)
members and the Government of India was signed in January, 2021 to strengthen mutual trust and
cooperation & enhance global energy security, stability and sustainability. This partnership will lead
to an extensive exchange of knowledge and would be a stepping stone towards India becoming a full
member of the IEA.
• The contents of the Strategic partnership will be jointly decided by the IEA Members and India, including
a phased increase in benefits and responsibilities for India as an IEA Strategic partner, and building on
existing areas of work within the Association and the Clean Energy Transitions Programme (CETP), such
as Energy Security, Clean & Sustainable Energy, Energy Efficiency, Enhancing petroleum storage
capacity in India, Expansion of gas-based economy in India etc.
• The IEA Secretariat will be responsible for the implementation of the cooperative activities in India and
for facilitating discussion between the IEA Members and India to further develop the Strategic
Partnership.
• The Government of India endeavours to take the necessary steps to encourage and promote strategic and
technical co-operation in the energy sector in the identified areas noted above, through the Framework
Agreement.
• Hence, option (d) is the correct answer.

Q 14.D
• Laschamp Excursion: It was a geomagnetic excursion i.e a short reversal of the Earth’s magnetic
field. It occurred during the end of the Last Glacial Period around 41,400 years ago.
o This event is named after the village where it was discovered in the French Massif Central.
o It led to series of catastrophic events like the destruction of ozone layer, electrical storms raged across
the tropics, solar winds generated spectacular light shows (auroras), Arctic air poured across North
America, ice sheets and glaciers surged and weather patterns shifted violently.
o During these events, life on earth was exposed to intense ultraviolet light, Neanderthals and giant
animals known as megafauna went extinct, while modern humans sought protection in caves.
o As per the recent finding of a new multidisciplinary study, the world experienced a few centuries
of apocalyptic conditions 42,000 years ago, triggered by a reversal of the Earth’s magnetic poles
combined with changes in the Sun’s behaviour.
• Hence, option (d) is the correct answer.

Q 15.C
• Recent Context: Recently, the Nepal government published its new political map claiming certain
areas as a part of its territory which are territorial issues between India and Nepal.
• Kalapani: It is a region located in the easternmost corner of Uttarakhand’s Pithoragarh
district. It shares a border on the north with the Tibet Autonomous Region of China and Nepal in
the east and south.
• The Kalapani region derives its name from the river Kali. Nepal’s claims to the region is based on this
river as it became the marker of the boundary of the kingdom of Nepal following the Treaty of
Sugauli signed between the rulers of Kathmandu and the East India Company after the first Anglo-Nepal
War (1814-16).
• According to Article 5 of the treaty, the King of Nepal gave up his claims over the region west of the
river Kali which originates in the High Himalayas and flows into the great plains of the Indian
subcontinent. Nepal’s claim of the territory east of Kali is based on the Limpiyadhura origin, India
says the river actually takes the name Kali near Kalapani.
• Therefore, it becomes territorial issues between both nations over Limpiyadhura Kalapani
and Lipulekh (trijunction between Nepal-India and China )
• In May 2020, India inaugurated the Darchula-Lipulekh pass link road, cutting across the disputed
Kalapani area which is used by Indian pilgrims to Kailash Mansarovar.
• Susta region (Tarai region is west Bihar) is another area of dispute between both nations.

6 www.visionias.in ©Vision IAS

DEEPAK PHOTOSTAT 9310521834


• Therefore, from the map, it can be seen that the current sequence of regions from west to East is:
Limpiyadhura – Kalapani- Lipulekh. Hence option (c) is the correct answer.

Q 16.B
• The phenomenon of loud noises most similar to that of thunder or a cannon being shot is termed as
Seneca Guns. It may induce tremors on land similar to earthquakes. These sounds are often heard in the
coastal areas and they are absent in the seas. Scientists have recently found a method to pinpoint where
the explosions are coming from and what causes the sound. The findings were presented at the recently
held American Geophysical Union. Hence, option (b) is the correct answer.
o They are also called Skyquakes. It has also been reported in the banks of river Ganges in Himachal
Pradesh. The places where the Seneca Guns have been reported are Japan, Australia, Italy, Norway,
Brazil, Mexico, Uruguay, Myanmar, and Java Islands of Indonesia.
o Till date, the reason for occurrence is unknown. It derives its name from the Lake Seneca in New
York that witnessed Civil War battle and gun sounds are very commonly heard.
o These sounds have been heard prominently in the regions around North Carolina. They can very loud
and have impacts like shaking and rattling of houses, damage to window panes etc.
o Researchers have stated that the sound could be originated from fast approaching outer space object’s
explosion once it enters the earth’s atmosphere or due to crash of ocean waves. But none of the
theories have been scientifically proved.

Q 17.D
• As per the existing system, the judges of Supreme Court and High Court are appointed by the President on
the recommendation of Collegium.
• The constitutional provisions related to their appointment are:
o Article 124 says the President should appoint Supreme Court judges after consultation with such
judges of High Courts and the Supreme Court as he/she may deem necessary. The Chief Justice of
India is to be consulted in all appointments barring his/her own.
o Article 217, which deals with the appointment of High Court judges, says the President should consult
the CJI, Governor, and Chief Justice of the High Court concerned.
o Neither of the above articles speak about collegium system
• The Chief Justice of India should consult a collegium of four senior most judges of Supreme Court
for appointment of judge to Supreme Court. In case of appointment of high court judge, the Chief
Justice of India should consult the collegium of two senior most judges of Supreme Court. The President
of India, as advised by the government, can only ask the collegium to reconsider its recommendation once
but if the collegium reiterates its recommendation the President will be bound to accept the same. The
term collegium system is not mentioned in the constitution. Hence statement 1 is not correct.

7 www.visionias.in ©Vision IAS

DEEPAK PHOTOSTAT 9310521834


• The appointments of the judges are formally made by the President of India on the
recommendation of the collegium and the proposals are processed through the Ministry of Law and
Justice, Government of India. As no timeline is prescribed by the Supreme Court in its judgments
during which the central government is bound to clear the names of recommended persons as judges, this
lacuna helps the central government to delay the judicial appointments if the government does not like
such persons recommended by the collegium. The government cannot refuse to accept the
recommendation of the collegium but it can certainly delay the appointment of the recommended person
by delaying in clearing the names of recommended persons as judge. Hence statement 3 is not correct.
• Sometimes there is lack of consensus in collegium resulting in stalemate. It leads to deadlock in collegium
which delays in recommending names of persons to be appointed as judge. It leads to delay in
appointment of judges. Recently it has been reported that the collegium, which is headed by CJI SA
Bobde, has met at least three times in last one month to decide on names of judges for elevation to
the apex court, but each time, it has been unable to decide on any name simply because of the lack
of consensus. Hence statement 2 is not correct.

Q 18.B
• Megasthenes was an ancient Greek historian, diplomat and Indian ethnographer. Megasthenes was the
ambassador who was sent to the court of Chandragupta Maurya by the Greek ruler of West Asia
named Seleucus Nicator. He described India in his book Indika, which is now lost, but has been partially
reconstructed from literary fragments found in later authors. Hence statement 1 is correct.
• The Second Buddhist Council met at Vaishali a century after Buddha’s death. The Buddhist Order split
into two later. One was called the Sthaviravadins or ‘Believers in the Teachings of the Elders’ and the
other known as Mahasanghikas or ‘Members of the Great Community’. The Third Buddhist Council
was held at Pataliputra. It was convened by Asoka. The Sthaviravadins established themselves
strongly and expelled the heretics. The last section called “Kathavatthu” was added to Abhidhamma
Pitaka. Hence statement 2 is not correct.
• There were five major political centres in the empire – the capital Pataliputra and the provincial centres of
Taxila (North), Ujjayini (West), Tosali (East) and Suvarnagiri (South), all mentioned in the Ashokan
inscriptions. These centres were carefully chosen, both Taxila and Ujjayini being situated on important
long-distance trade routes, while Suvarnagiri (literally, the golden mountain) was possibly important
for tapping the gold mines of Karnataka. Hence statement 3 is not correct.
• The Great Stupa at Sanchi is one of the oldest stone structures in India. It was originally commissioned
by the emperor Ashoka in the 3rd century BCE. It was built with bricks during the time of Ashoka and
later it was covered with stone. Hence statement 4 is correct.

Q 19.B
• The Kaipad system of rice cultivation is an organic farming system in which rice cultivation and
aquaculture go together in coastal brackish water marshes. Chemical fertilizers are not used in rice, fish
and shrimp farming in the fields.
• Kaipad farming is done in Kozhikode, Kannur and Kasaragod districts on the Malabar coast.
Hence, option (b) is the correct answer.
• It was almost abandoned by farmers owing to a labour shortage and huge production cost. However, due
to government initiatives, it is on a revival path.
• Kaipad rice received Geographical Indication tag in 2014.

Q 20.B
• The Tree Cities of the World programme is an international effort to recognize cities and towns
committed to ensuring that their urban forests and trees are properly maintained, sustainably managed,
and duly celebrated.
• Tree Cities of the World programme aims to connect cities around the world in a new network dedicated
to sharing and adopting the most successful approaches to managing community trees and forests.
• To be recognized as a Tree City, a community must meet five core standards that illustrate a commitment
to caring for its trees and forest:
o STANDARD 1: Establish Responsibility The city has a written statement by city leaders delegating
responsibility for the care of trees within the municipal boundary to a staff member, a city department,
or a group of citizens—called a Tree Board.
o STANDARD 2: Set the Rules The city has in place a law or an official policy that governs the
management of forests and trees. These rules describe how work must be performed—often citing

8 www.visionias.in ©Vision IAS

DEEPAK PHOTOSTAT 9310521834


best practices or industry standards for tree care and worker safety—where and when they apply, and
penalties for noncompliance.
o STANDARD 3: Know What You Have The city has an updated inventory or assessment of the local
tree resource so that an effective long-term plan for planting, care, and removal of city trees can be
established.
o STANDARD 4: Allocate the Resources The city has a dedicated annual budget for the routine
implementation of the tree management plan.
o STANDARD 5: Celebrate Achievements The city holds an annual celebration of trees to raise
awareness among residents and to acknowledge citizens and staff members who carry out the city tree
programme.
• There is no criterion for a city to have 33 per cent of its geographical area to be eligible to become
Tree city of the World. Hence, statement 1 is not correct.
• It is a programme of the Arbor Day Foundation and the Food and Agriculture Organisation (FAO).
Hence, statement 2 is correct.
• Recently Hyderabad has become the only city in India to be recognized as a ‘Tree City of the
World’.
• United States, Canada, and the United Kingdom are the countries with the maximum cities featured on the
list, with 38, 15, and 11 cities, respectively.

Q 21.A
• Inflation is the decline of purchasing power of a given currency over time. A quantitative estimate of
the rate at which the decline in purchasing power occurs can be reflected in the increase of an average
price level of a basket of selected goods and services in an economy over some period of time.
o The rise in the general level of prices often expressed a percentage means that a unit of currency
effectively buys less than it did in prior periods. As a currency loses value, prices rise and it buys
fewer goods and services.
• Most commonly used inflation indexes are the Consumer Price Index (CPI) and the Wholesale Price
Index (WPI). Inflation can be viewed positively or negatively depending on the individual viewpoint
and rate of change.
• A progressive tax is a tax in which the tax rate increases as the taxable amount increases. Hence,
statement 1 is correct.
• A regressive tax is a tax applied uniformly, taking a larger percentage of income from low-income earners
than from high-income earners. A regressive tax affects people with low incomes more severely than
people with high incomes because it is applied uniformly to all situations, regardless of the taxpayer.
• Inflation operates like a regressive consumption tax because the extent to which individuals are
affected by the inflation tax depends on their level of consumption. An increase in inflation raises the
cost of transacting for individuals at the bottom of the income distribution to a greater extent than for
those at the top. For example, consider person A with low income and person B with High income both
buying a product whose price has increased due to inflation. Now both A and B have to spend extra
money to purchase the product. But the extra spending forms a higher percentage share in the income of A
compared to the high income of B. Therefore, Inflation is a regressive tax. Hence statement 2 is not
correct.

Q 22.B
• Similipal National Park is a national park and a tiger reserve in the Mayurbhanj district in the
Indian state of Odisha.
o It is part of the Mayurbhanj Elephant Reserve, which includes three protected areas — Similipal
Tiger Reserve, Hadgarh Wildlife Sanctuary and Kuldiha Wildlife Sanctuary.
o Similipal National Park derives its name from the abundance of red silk cotton trees growing in the
area.
o This protected area is part of the UNESCO World Network of Biosphere Reserves since 2009
o The park is home to Bengal tiger, Asian elephant, gaur, chausingha etc. along with some of the
waterfalls like Joranda and Barehipani Falls.
o The vegetation of the Similipal National Park ranges from Semi-Evergreen to Dry Deciduous.
o While forest fires tend to be a common occurrence in Similipal, the latest fire event is unprecedented
in terms of its intensity. Moreover, the early onset of summer this year and a prolonged dry spell in
the region have further contributed to the wildfire spread in this reserve.
o Also, the period from January-April coincides with the Akhand Shikar ritual of many forest-
dwelling communities during which, the Adivasis usually clear the ground bushes and grasses by
9 www.visionias.in ©Vision IAS

DEEPAK PHOTOSTAT 9310521834


setting them on fire to enhance their visibility in the forest. Among others, this is said to be one of the
causes of forest fires in the Simlipal national park.
• Hence option (b) is the correct answer.

Q 23.B
• United Nations Development Programme (UNDP) recently released the Human Development
Report, 2020.
• Usually, the UNDP uses three major parameters to calculate the Human Development Index based on
which the countries are ranked. They are Life Expectancy at birth, maternal mortality rate and political
participation of women.
• This year, in 2020, for the first time the UNDP has introduced Planetary Pressures-adjusted HDI
(PHDI). However, it is an experimental metric that adjusts the Human Development Index (HDI)
for planetary pressures. It has not been yet adopted in calculating the HDI.
o It is the level of human development adjusted by carbon dioxide emissions per person and
material footprint per capita. The material footprint is the amount of fossil fuels, metals and other
resources consumed by a country to produce the required Goods and Services. Hence option (b) is
the correct answer.
o In an ideal scenario where there are no pressures on the planet, the PHDI equals the HDI. However,
all countries of the world have substantial pressure from their part.
o If this new metric is taken into account, Ireland tops the list while India moves 8 positions up from the
HDI rank of 131.

Q 24.C
• Under the Central Goods and Services Tax (CGST) Act, 2017, Government has set up the
Consumer Welfare Fund (CWF) to promote and protect the welfare of the consumers. The Consumer
Welfare Fund Rules have since been incorporated in Rule 97 of the CGST Rules, 2017. Hence, statement
2 is not correct.
• The overall objective of the Consumer Welfare Fund is to provide financial assistance to promote
and protect the welfare of the consumers and strengthen the consumer movement in the country.
Hence, statement 1 is correct.
• The Fund has been set up by the Department of Revenue and, is being operated by the Department
of Consumer Affairs, Ministry of Consumer Affairs, Food & Public Distribution. Hence, statement
3 is correct.
• Financial assistance from CWF is given to various Institutions including Universities, Voluntary
Consumer Organization (VCOs) and States to promote and protect the welfare and interests of the
consumers, create consumer awareness and strengthen consumer movement in the country.
• A Standing Committee headed by the Secretary, Department of Consumer Affairs has been constituted to
make recommendations for proper utilization of the money credited to the Consumer Welfare Fund for the
welfare of consumers.

Q 25.D
• The landholders' society the first political association of modern India was founded in 1837-38 at
Calcutta. The basic purpose was to promote and protect the class interests of landlords and zamindars.
Its zone of influence was limited to erstwhile Bengal province only. Hence statements 1 and 2 are
correct.
• The British Indian Association was founded on October 29, 1851, at Calcutta. The Landholders' society
of 1837 and Bengal British India society of 1843 later merged with the British Indian association in
1851. Raja Radhakant Deb who was associated with erstwhile Bengal landholders society assigned
the position of president in the merged body of British Indian association with Debendra Nath
Tagore as its secretary. Hence statement 3 is correct.

Q 26.B
• The Galwan River flows from the disputed Aksai Chin region administered by China to the Ladakh
region of India.
• It originates on the eastern side of the Karakoram range and flows west to join the Shyok River. The
point of confluence is around 100 km south of Daulat Beg Oldi.
o Daulat Beg Oldi is a camp site and current military base located in the midst of the Karakoram Range
in northern Ladakh
• Shyok River itself is a tributary of the Indus River, making Galwan a part of the Indus River system.
10 www.visionias.in ©Vision IAS

DEEPAK PHOTOSTAT 9310521834


• The Galwan River forms a narrow valley as it flows through the Karakoram mountains, which has been a
bone of contention in their border dispute and led to a recent faceoff between China and India.
• In 1962, a forward post set up by India in the upper reaches of the Galwan Valley which was contested
and ultimately dismantled during 1962 war.
• Hence option (b) is the correct answer.

Q 27.B
• Recent Context - Recently, the UN High Commissioner for Human Rights has raised concern over the
misuse Of Sedition Law in India. While referring to the farmers protest in India she said that "Charges of
sedition against journalists and activists for reporting or commenting on the protests, & attempt to curb
freedom of expression on social media are disturbing departures from essential human rights principles".
• Section 124A in Chapter VI of Indian Penal Code deals with the Offence ‘Sedition’. Chapter VI deals
with ‘Offences against the State'.
• The Section 124-A reads as follows; “Whoever by words, either spoken or written, or by signs, or by
visible representation, or otherwise, brings or attempts to bring into hatred or contempt, or excites or
attempts to excite disaffection towards, the Government established by law in India, shall be punished
with imprisonment for life, to which fine may be added, or with imprisonment which may extend to three
years, to which fine may be added, or with fine”.
• Section 124A was not part of the Original Indian Penal Code of 1860. It was incorporated into the code in
the 1870s, as a response to the rising Wahabi movement.
• The Fundamental Rights Sub-Committee of the Constituent Assembly expressly included sedition as
grounds for restricting free speech in its first draft of the fundamental rights. The Constituent Assembly
objected it strongly. When Article 19(1)(a) finally came into being, “sedition" was not among the
permissible restrictions under Article 19(2). Hence statement 1 is not correct.
• Sedition in India is a cognizable (not requiring a warrant for an arrest), non-compoundable (not allowing a
compromise between the accused and the victim), and non-bailable offence. The penalty can range from a
fine to three years or life imprisonment. But these penalties would be awarded after the judgement, which
can take a long while to come. Meanwhile, a person charged with sedition must live without their
passport, barred from government jobs, and must produce themselves in the court on a whenever
required. Hence statement 2 is correct.
• Many Indian freedom fighters including Mahatma Gandhi and Bal Gangadhar Tilak were charged with
Sedition during freedom Struggle. The first case in India that arose under this section is known as the
Bangobasi case (Queen-Empress v. Jagendra Chunder Bose). Bal Gangadhar Tilak was convicted under
this Section and sent to Mandalay, Burma from 1908 to 1914 for defending the Indian revolutionaries and
called for immediate Swaraj or self-rule in his newspaper ‘Kesari’ (Queen Empress Vs. Bal Gangadhar
Tilak (1897)). In 1959 Allahabad High Court declared that Section 124A of IPC was ultra vires to Art.
19(1)(a) of the Constitution. The above decision was overruled by the Supreme Court in Kedar Nath Case
• Statement 3 is not correct: Some of the famous sedition trials during freedom movement included
numerous prosecutions against the editors of nationalist newspapers. The first among them was the trial
of Jogendra Chandra Bose in 1891. Bose was the editor of the newspaper Bangobasi and wrote an
article criticizing the Age of Consent Bill for posing a threat to the religion and for its coercive
relationship with Indians. The most well-known cases are the three sedition trials of Bal Gangadhar
Tilak and the trial of Mahatma Gandhi in 1922. Gandhi was charged along with Shankerlal Banker, the
proprietor of Young India, for three articles published in the weekly.

Q 28.A
• Shivaji was also a great administrator. He laid the foundations of a sound system of administration. The
king was the pivot of the government. He was assisted by a council of ministers
called Ashtapradhan. However, each minister was directly responsible to Shivaji.
o Peshwa – Finance and general administration. Later he became the prime minister.
o Sar-i-Naubat or Senapati – Military commander, a honorary post.
o Amatya – Accountant General.
o Waqenavis – Intelligence, posts and household affairs.
o Sachiv – Correspondence.
o Sumanta – Master of ceremonies.
o Nyayadish – Justice.
o Panditarao – Charities and religious administration. Hence option (a) is the correct answer.
• Most of the administrative reforms of Shivaji were based on the practices of the Deccan sultanates. For
example, Peshwa was the Persian title. The revenue system of Shivaji was based on that of Malik Amber
11 www.visionias.in ©Vision IAS

DEEPAK PHOTOSTAT 9310521834


of Ahmadnagar. Lands were measured by using the measuring rod called Kathi. Lands were also
classified into three categories – paddy fields, garden lands and hilly tracks. He reduced the powers of the
existing deshmuks and kulkarnis. He appointed his own revenue officials called Karkuns.

Q 29.B
• The Shimla Conference 1945, was held to approve the Wavell Plan named after the Viceroy Lord Wavell
for Indian self-government. It constituted the resolution of political deadlocks ( Muslim League vs. Indian
National Congress) of India during British Rule. It proclaimed a plan for a new Executive Council in
which all the members except the Viceroy and Commander in chief would be Indians. Hence statement 1
is not correct.
• It proposed an interim government plan in which all the portfolios except the DEFENCE would be held
by Indian members. Hence statement 2 is correct.
• M. A. Jinnah attended the conference on behalf of the Muslim League. He made a strange claim that
Congress had no right to nominate any Muslim in the Executive Council. This demand was unacceptable
to the Congress and Muslim League was adamant. Thus ultimately Lord Wavell himself dropped the
plan. Hence statement 3 is not correct.

Q 30.D
• 73rd Constitutional Amendment Act (1992) or the Part IX of the Constitution provides for:
o Direct elections to all seats in panchayats at the village, intermediate and district levels.
o Indirect elections to the post of chairperson of panchayats at the intermediate and district levels by
and from amongst the elected members thereof.
o However, the chairperson of a panchayat at the village level shall be elected in such manner as the
state legislature determines.
• General Elections to the three tiers of Panchayats are held simultaneously. Each voter will cast his votes,
in the same polling station, for choosing the members for election to Gram Panchayat, Panchayat Samiti
and Zilla Parishad. Hence, statement 1 is correct.
• Within this three-tier panchayat system, unlike the higher tiers (Taluk and Zilla Panchayats), the elections
to grama panchayats (GPs) are being held on the basis of non-political party symbols. In other words,
candidates contesting for grama panchayats cannot use the symbols of recognized political (national and
local) parties. Governments may choose to conduct Taluk and Zilla Panchayats election with or without
without the symbols of political parties. Hence, statement 2 is correct.
• One of the major reasons for adopting such ‘party-less’ election is to avoid poll related conflicts both
during and after the election. The argument for this is that since villages are societies where every villager
knows the other by face and lives in harmony and cooperation. Political party based elections to grama
panchayats could lead to divisions within the community on party lines, with hatred, criminalization, and
segregation emerging between the castes and classes. These will ultimately hamper local governance and
development.
• The Model Code of Conduct comes into force immediately on the announcement of the election schedule
by the election commission for the need of ensuring free and fair elections. Its main purpose is to ensure
that contesting candidates and parties do not resort to unfair means to gain an unfair edge. Model Code of
Conduct for the Guidance of Political parties and candidate in elections to Panchayats and Municipality is
issued by the respective State Election Commission. Hence, statement 3 is correct.

Q 31.A
• The Home Rule League was the Indian response to the First World War in an aggressive manner. The
people were facing a burden of the wartime miseries after the first world war, This motivated the masses
to engage in a proactive struggle against the British odds. Hence statement 1 is correct.
• The world war being fought amongst the major imperial powers was backed by the propaganda of selfish
expansion exposed the myth of white superiority which was being narrated in British colonies since late
18th century. Hence statement 3 is correct.
• The league was set up after Tilak and Besant failed to reach a moderate-extremist rapprochement. Annie
Besant also failed to get the Congress to approve her scheme of Home Rule Leagues, as the Congress did
not respond to her demands and commitments in the initial days of the league. Hence statement 2 is not
correct.

12 www.visionias.in ©Vision IAS

DEEPAK PHOTOSTAT 9310521834


Q 32.C
• Mica is a mineral of strategic importance because of its perfect cleavage, flexibility, infusibility, high heat
and electrical insulation and high dielectric strength, a combination of qualities not possessed by any other
natural material.
• The chief demand for mica comes from the electrical industry, while in powder form after grinding to a
suitable size, it is also used as a 'filler in various industries.
• In 2015 mica has been declared as a minor mineral. Hence statement 1 is correct.
• India is one of the important sources of sheet (block) mica producing between 70 and 80% of the total
block mica output of the world.
• The splitting of mica is done by hand.
• The mica industry of the country depends largely on the export market, the domestic consumption being
small.
• Commercially, two varieties of mica are important—
o Muscovite or potash mica and phlogopite or magnesian mica.
o Granitic pegmatites are the source of the muscovite sheet.
o Phlogopite is found in areas of metamorphosed sedimentary rocks into which pegmatite rich granite
rocks have been intruded.
o Most of the deposits in India produce the muscovite variety, phlogopite is available in comparatively
small quantities from a few places mainly in Travancore. Hence statement 3 is correct.
o Both the varieties are used for electrical insulation, the best qualities being used for high tension
electrical work.
• The most important mica-bearing pegmatites occur in Andhra Pradesh, Bihar, Jharkhand, Maharashtra,
Odisha, Rajasthan and Telangana.
• Occurrences of mica pegmatites are also reported from Gujarat, Haryana, Karnataka, Kerala, Tamil Nadu
and West Bengal.
• Andhra Pradesh has the highest reserve of mica and it is also the largest mica producing state of
India. Hence statement 2 is not correct.

Q 33.B
• Tagore’s achievement in literary and creative spheres equals Gandhi’s achievement in political sphere.
Both of them exhibited the urge to put India in world literary and political thought.
Gandhi visited Shantiniketan on 10th March 1915. To this day it is observed as ‘Gandhi punyaha Din’
at Shantiniketan every year. It is during February 1915 that Tagore referred Gandhi as Mahatma.
• Gandhi’s ‘Village Swaraj’ and Tagore’s ‘Swadeshi Samaj’ had a common meeting ground and both
believed that India’s domination by foreign rule was self-made and could be challenged by the soul-force.
In 1934 Gandhi had become ‘Advisor’ to the Village Industries Association in Shantiniketan at Tagore’s
request.
• There were differences between these luminaries, the foremost being the issue of non cooperation
movement which Gandhi launched. Tagore believed that the idea of non cooperation is political
asceticism. It may develop into frightfulness in the human nature, losing faith in the basic reality of
life. Hence statement 1 is not correct.
• Secondly, Tagore raised the issue of Charkha, in his essay, ‘Cult of Charkha’ (1925). Gandhi
propounded his Constructive Programme for rural development and economic regeneration through
propagation of Charkha. Tagore expressed apprehensions about not only its economic efficacy, but also
the use of moral language in place of the economic. He criticised the undue emphasis and blind faith in
Charkha that distracted attention from other more important factors in the task of all- round
reconstruction. Hence statement 2 is correct.
• Hence option (b) is the correct answer.

Q 34.C
• The provisions of Fundamental Rights, Directive Principles of State Policy and Fundamental Duties are
under the Part III, Part IV and Part IV-A of the Constitution respectively. The Fundamental Duties in the
Indian Constitution are inspired by the Constitution of erstwhile USSR. The Fundamental Duties of the
citizens are enumerated in A-51A. Fundamental Duties did not form part of the Constitution as originally
adopted. The duties are addressed to the citizens only and do not extend to foreigners. The fundamental
duties are 11 in number. Ten duties were included in the Indian Constitution by the 42nd Amendment Act
1976. The Eleventh duty was added by the 86th Amendment Act, 2002.

13 www.visionias.in ©Vision IAS

DEEPAK PHOTOSTAT 9310521834


• List of Fundamental Duties
o 51A (a) To abide the Constitution and respect its ideals and institutions, the National Flag and the
National Anthem
o 51A (b) To cherish and follow the noble ideals which inspired our national struggle for freedom
o 51A (c) To uphold and protect the sovereignty, unity and integrity of India
o 51A (d) To defend the country and render national services when called upon to do so
o 51A (e) To promote harmony and the spirit of common brotherhood amongst all the people of
India transcending religious, linguistic and regional or sectional diversities; to renounce
practices derogatory to the dignity of women. Hence option (c) is the correct answer.
o 51A (f) To value and preserve the rich heritage of our composite culture
o 51A (g) To value protect and improve the natural environment including forests, lakes, rivers and wild
life, and to have compassion for living creatures
o 51A (h) To develop the scientific temper, humanism and spirit of inquiry and reform
o 51A (i) To safeguard public property and to abjure violence
o 51A (j)To strive towards excellence in all spheres of individual and collective activity so that the
nation constantly rises to higher levels of endeavor and achievement
o 51A (k) Duty of the parent or guardian to provide opportunities for education to his child, as the case
may be, ward between the age of six and fourteen years (added by 86th Amendment Act, 2002)

Q 35.C
• What is a clone? A clone is a living organism (such as a plant or animal), which shares the same genetic
information as another organism. However, their characteristics can be affected by random mutations
which occur in their DNA during development in the womb or by the environment that they grow up in,
so, although clones have the same DNA, they may not look the same or behave in the same way.
• Hand-guided cloning technique: Traditional cloning (TC) necessitate the use of expensive
micromanipulators for enucleation of recipient oocytes, followed by insertion of a donor somatic cell or
nucleus into enucleated oocyte by fusion or direct injection besides requiring skilled expertise. One of the
major necessities in TC was to reduce the costs without compromising with the efficiency.
• Keeping this in view, some adaptations were made in TC to make it more robust and inexpensive, which
ultimately led to the birth of handmade cloning (HMC), a technique also known as zona free cloning or
hand guided technique. In this technique the oocytes isolated from abattoir ovaries were matured in
vitro, denuded, treated with an enzyme to digest the zona and then enucleated with the help of hand
held fine blade. Based on SCNT, HMC is advanced procedure of enucleation of zona-free mammalian
oocytes by hand-bisection using a metal blade or by employing density gradient centrifugation or by
chemicals. Hence, statement 1 is correct.
• The National Dairy Research Institute, Karnal has developed a high yield breed buffalo calf
"Tejas" using the hand guided cloning technology. The male calf named 'Tejas' is the clone of the high
yield buffalo breed. The new clone Tejas is of the Murra breed of buffaloes which are known for their
high milk yield. Hence statement 2 is correct. In 2009, the NDRI had produced the first female buffalo
clone named Garima at the institute. Garima then gave birth to Mahima and Karishma in 2013 and 2014
through normal birth.
Q 36.D
Fishing cat:
• Scientific name: Prionailurus viverrinus the fishing cat is an adept swimmer and enters water frequently
to prey on fish. Wetlands are the favourite habitats of the fishing cat and It is the only wild cat species in
India that is a wetland specialist. Hence statement 1 is correct.
• In India, fishing cats are mainly found in the mangrove forests of the Sundarbans, on the foothills of the
Himalayas along the Ganga and Brahmaputra river valleys and in the Western Ghats. It also has a patchy
distribution along the Eastern Ghats. Hence statement 3 is not correct.
• IUCN Red List: Vulnerable. Despite multiple threats, the Fishing Cat was recently downlisted to
“Vulnerable” from “Endangered” in the IUCN Red List species assessment. Hence statement 2 is
correct.
• CITES lists the fishing cat in Appendix II part of Article IV of CITES.
• In India, the fishing cat is included in Schedule I of the Indian Wildlife (Protection) Act, 1972 and thereby
protected from hunting.
• It is nocturnal (active at night) and apart from fish also preys on frogs, crustaceans, snakes, birds, and
scavenges on carcasses of larger animals.
• It is the state animal of West Bengal.
14 www.visionias.in ©Vision IAS

DEEPAK PHOTOSTAT 9310521834


Q 37.A
• Suture Zone:
o A suture zone is a linear belt of intense deformation, where distinct terranes or tectonic units with
different plate tectonic, metamorphic, and paleogeographic histories join together.
o The suture zone is often represented on the surface by an orogen or mountain range comprising
intensely deformed rocks.
• The Tuting-Tidding Suture Zone (TTSZ):
o It is a major part of the Eastern Himalaya, where the Himalaya takes a sharp southward bend and
connects with the Indo-Burma Range.
o This is also known as the eastern syntaxial bend, where the Brahmaputra river takes a hair-pin type
bend while entering India. Hence option (a) is the correct answer.
o This part of the Arunachal Himalaya has gained significant importance in recent times due to the
growing need for constructing roads and hydropower projects, making the need for understanding the
pattern of seismicity in this region critical.
• Indus Suture Zone:
o It is in the Ladakh region where Indian and Asian Plates are joined has been found to be tectonically
active, as against the current understanding that it is a locked zone.

Q 38.C
• Recent Context – Recently, Supreme Court Bench headed by Chief Justice of India (CJI) S A Bobde
objected to a petitioner addressing judges as “Your Honour”. It once again triggered debate around court
etiquette in India.
• The Advocates Act of 1961, under section 49(1)(c), empowers the Bar Council of India to make rules
on professional and etiquette standards to be observed by advocates.
• The Bar Council of India is a statutory body created by Parliament to regulate and represent the
Indian bar. It performs the regulatory function by prescribing standards of professional conduct
and etiquette and by exercising disciplinary jurisdiction over the bar. It also sets standards for legal
education and grants recognition to Universities whose degree in law will serve as qualification for
enrolment as an advocate.
• The Bar Council of India was established by Parliament under the Advocates Act, 1961. The following
statutory functions under Section 7 cover the Bar Council’s regulatory and representative mandate for the
legal profession and legal education in India:
o To lay down standards of professional conduct and etiquette for advocates. Hence option (c) is
the correct answer.
o To lay down procedure to be followed by its disciplinary committee and the disciplinary committees
of each State Bar Council.
o To safeguard the rights, privileges and interests of advocates.
o To promote and support law reform.
o To deal with and dispose of any matter which may be referred to it by a State Bar Council.
o To promote legal education and to lay down standards of legal education. This is done in consultation
with the Universities in India imparting legal education and the State Bar Councils.
o To recognise Universities whose degree in law shall be a qualification for enrolment as an advocate.
The Bar Council of India visits and inspects Universities, or directs the State Bar Councils to visit and
inspect Universities for this purpose.
o To conduct seminars and talks on legal topics by eminent jurists and publish journals and papers of
legal interest.
o To organise legal aid to the poor.
o To recognise on a reciprocal basis, the foreign qualifications in law obtained outside India for the
purpose of admission as an advocate in India.
o To manage and invest the funds of the Bar Council.
o To provide for the election of its members who shall run the Bar Councils.

Q 39.B
• Vermicomposting is one of the sustainable agriculture practices that follow the principles of organic
farming. It is a natural organic fertilizer prepared by using earthworms that convert organic
compounds into manure.
• During vermicomposting organic matter is decomposed into ammonia which neutralizes the acid
components in the soil shifting to alkaline pH. This improves the fertility of the soil. Hence statement
1 is correct.
15 www.visionias.in ©Vision IAS

DEEPAK PHOTOSTAT 9310521834


• In vermicomposting system, earthworms act as voracious feeder, modifying the composition of organic
waste, gradually reducing its organic carbon and C: N ratio and retains more nutrients (nitrogen,
potassium, phosphorus, calcium). The nutrient content is generally higher in vermicompost than in
traditional compost. Hence statement 2 is correct.
• Earthworm increases the surface area of any material and makes it more favourable for the activity of
microbiota for further decomposition. It is an environmentally friendly process that converts
biodegradable matter into vermicast. Vermicast is rich in nutrients that help in plant growth and inhibits
harmful microbes in the soil. Thus it is called black gold.
• Advantages of vermicompost
o Vermicompost helps improve soil structure, texture, porosity, water holding capacity, drainage, and
aeration and reduce erosion.
o It improves plant growth by enabling the growth of new shoots and leaves, thereby increasing
productivity.
o It helps to neutralize the pH of the soil.
o Vermicompost enhances the soil’s microbial activity and adds beneficial microbes, lowers the pest
and disease incidence.
o It is easy to apply and handle, economical and does not have a foul odour.
o It is free from any pathogens or harmful materials.
o It contains several micro and macronutrients, vitamins, enzymes and hormones like auxins and
gibberellins.
• Disadvantages of Vermicompost
o Odour- If not done properly, it releases a bad odour. Filling the compost bin with green plants forms
ammonia and produces the smell. Adding carbon sources like paper helps to neutralize the smell. The
bad odour attracts rodents and flies. So, the bin should be covered properly.
o Time- It is a time-consuming process and usually takes 2-3 months for completing the process
o Maintenance- Temperature needs to be maintained for better action by earthworms. The net
worm biomass achieved is higher during winter than the summer season. The temperature variations
during winter support the life activities of earthworms more favourably than in summer. Hence
statement 3 is not correct.

Q 40.B
• The central government has approved a proposal to levy ‘Green Tax’ on old vehicles which are
considered to be polluting the environment. The main principles to be followed while levying the Green
Tax are:
o Transport vehicles older than 8 years could be charged Green Tax at the time of renewal of
fitness certificate, at the rate of 10 to 25 % of road tax;
o Personal vehicles to be charged Green Tax at the time of renewal of Registration Certification
after 15 years; Hence, statement 1 is not correct.
o Public transport vehicles, such as city buses, to be charged lower Green tax;
o Higher Green tax (50% of Road Tax) for vehicles being registered in highly polluted cities
o Differential tax, depending on fuel (petrol/diesel) and type of vehicle;
o Vehicles like strong hybrids, electric vehicles and alternate fuels like CNG, ethanol, LPG etc. to
be exempted;
o Vehicles used in farming, such as tractor, harvester, tiller etc. to be exempted; Hence, statement
2 is correct.
o Revenue collected from the Green Tax to be kept in a separate account and used for tackling
pollution, and for States to set up state of-art facilities for emission monitoring
• The benefits of the “Green Tax” could be:
o To dissuade people from using vehicles which damage the environment
o To motivate people to switch to newer, less polluting vehicles
o Green tax will reduce the pollution level, and make the polluter pay for pollution.
• It is estimated that commercial vehicles, which constitute about 5% of the total vehicle fleet, contribute
about 65-70% of total vehicular pollution. The older fleet, typically manufactured before the year 2000
constitute less that 1 % of the total fleet but contributes around 15% of total vehicular pollution. These
older vehicles pollute 10-25 times more than modern vehicles.

16 www.visionias.in ©Vision IAS

DEEPAK PHOTOSTAT 9310521834


Q 41.D
• Photonics is the science and technology of light. Photonics deals with generating, guiding, manipulating,
amplifying and detecting light. Lasers, optical fibres, the cameras and the displays in our phones, LED
lighting in our cars, homes, computer screens & TVs and optical tweezers are just a few examples of how
photonics have changed technology and affected our daily lives.
• The ability of light to detect and measure in a fast, sensitive and accurate way gives photonics a unique
potential to revolutionise healthcare. Biophotonics is the use of light-based technologies in the bio &
medical sciences. It can be used effectively for very early detection of diseases, with non-invasive
imaging techniques or point-of-care applications. Biophotonics is also instrumental to the analysis of
processes at the molecular level, giving a greater understanding of the origin of diseases, and hence
allowing prevention and new treatments.
• Lighting and Energy savings: Photonics are used also in advanced lighting technology, such as Solid
State Lighting (SSL) for general lighting applications. SSL is based on light emitting diodes (LEDs)
and organic LED (OLED) technologies. It provides higher quality lighting and contributes to substantial
energy savings. Intensive manufacturing and research activities aim to further improve SSL performance -
particularly energy efficiency and quality - and therefore reduce costs. Lighting adds up to 19% of
electricity consumption worldwide, thus more efficient lighting will provide huge energy savings. For
example, the replacement of incandescent bulbs with SSL technologies would save up to 70% of the
energy used today in lighting.
• Broadband Internet: It addresses the dramatic increase of power consumption on the web, in data
centres and servers. It allows traffic growth, fast network changes and varying traffic demands while
making data communications faster, cheaper and less energy consuming.
• Safety and security: Photonics is a key enabler for enhancing the safety and security of people, goods
and the environment. It makes possible to build contactless sensors and visual applications operating
in various ranges of the light spectrum (from X-ray to Terahertz), sufficiently sensitive and accurate
to reliably detect potential hazards or dangerous situations. Photonics technologies have several practical
applications in safety and security: from fibre sensors useful to detect structural defects in the building
sector, prevent environmental pollution or to develop driver assistance systems to security applications
like biometrics and border security systems, video surveillance systems and equipment to detect
dangerous or illegal goods. Hence option (d) is the correct answer.

Q 42.A
• Statement 1 is correct: A smart contract is a self-executing contract with the terms of the agreement
between buyer and seller being directly written into lines of code. The code and the agreements
contained therein exist across a distributed, decentralized blockchain network. The code controls the
execution, and transactions are trackable and irreversible.
• All smart contracts share some common properties.
o Because they exist on the blockchain, they have a state, like RAM in a computer does, and this state is
shared across the entire network. So, each node running this blockchain has a copy of the state of the
smart contract.
o They cannot be altered. Although there are ways to extend them or replace parts — if such action has
been foreseen by the developers — there is no way to covertly manipulate their content without
drawing the attention of the network.
o The logic of a smart contract cannot be distorted, so there is no room for interpretation.
o Smart contracts offer several benefits when they’re deployed appropriately. They can allow people
from around the globe to transact with one another without needing an intermediary, reducing the
costs of middlemen and brokers.
o Because there is no third party involved, there is no risk of manipulation. Smart contracts can reduce
administration, saving time. They offer complete autonomy, and because everything is backed up on
to the blockchain, smart contracts are completely safe against loss of data.
o Statement 2 is not correct: One significant challenge with smart contracts is that they’re
irreversible. If the code has bugs, then it could be that unwanted transactions occur and there is
currently no way to undo them.

Q 43.C
• Torrefaction is a low-temperature process of biomass and biosolids pyrolysis that leads to obtaining a dry
product with enhanced fuel properties (bio-coal, charcoal, torrefied biomass). Hence, statement 1 is
correct.

17 www.visionias.in ©Vision IAS

DEEPAK PHOTOSTAT 9310521834


• Torrefaction involves the heating of biomass in the absence of oxygen to a temperature of typically 250-
350°C (and not 800 degrees Celsius). The structure of the biomass changes in such a way, that the
material becomes brittle, and more hydrophobic, making grinding easier and less energy-intensive.
Although weight loss is about 30%, the energy loss is only 10%. Hence, statement 2 is not correct.
• The main product is solid, torrefied biomass. However, during the torrefaction process, combustible gas is
released, which is utilised to provide heat to the process. Hence, statement 3 is correct.
• Compared to fresh biomass, storage of the torrefied material can be substantially simplified since
biological degradation and water uptake is minimized.
• This changes the elements of the biomass into coal-like pellets. These pellets can be used for combustion
along with coal for industrial applications like steel and cement production

Q 44.A
• Martial law is mentioned (but not defined) under the Article 34 of Indian constitution. It is borrowed
in India from the English common law.
• Article 34 empowers the Parliament to indemnify any government servant or any other person for
any act done by him in connection with the maintenance or restoration of order in any area where
martial law was in force. Hence statement 1 is correct.
• The Act of Indemnity made by the Parliament cannot be challenged in any court on the ground of
contravention of any of the fundamental rights.
• During the operation of martial law, the military authorities are vested with abnormal powers to take
all necessary steps, however The Supreme Court held that the declaration of martial law does not
ipso facto (by that particular happening) result in the suspension of the writ of habeas
corpus. Hence statement 2 is not correct.
• Martial law has been imposed several times in India during its pre-independence period. Moreover, the
martial laws where enforced rightly at the time when there was a fight for freedom. The first such martial
law was imposed in 1817 in Cuttack, Odisha. After the above war, again the martial law was imposed in
1857, during Sepoy Mutiny, ought to be called the First War of Independence. Martial law has never been
imposed in India, post-independence till date. Hence statement 3 is correct.

Q 45.C
• Statement 1 is correct: Recently the scientists of Indian Council of Agriculture Research or ICAR, have
found a cure for one of the most dreaded agricultural diseases. The fungal disease, called Fusarium
Wilt, is popularly known as the ‘Panama Disease’ and afflicts banana plants. Fusarium wilt is a
widespread plant disease caused by many forms of the soil-inhabiting fungus Fusarium
oxysporum. Several hundred plant species are susceptible, including economically important food crops
such as sweet potatoes, tomatoes, legumes, melons, and bananas.
• This disease affects the Cavendish variety or the G9 Banana cultivar, which is the most widely grown
banana in the world. For the first time, Indian scientists have brought out a biopesticide that can control
the disease. This biopesticide has been made using another fungus.
• In India, more than 60 per cent of bananas are of the G9 variety. They go by names like ‘Grand Naine’,
‘Robusta’, ‘Bhusaval’, ‘Basrai’ and ‘Shrimanth’. Farmers in at least four Indian states — Bihar, Gujarat,
Madhya Pradesh and Uttar Pradesh — have been badly affected by this disease.
• Panama Disease is caused by a fungus with a long and complicated name called Fusarium Oxysporum f.
Sp cubense. One of its strains which is called ‘Tropical Race 4’ or ‘TR4’ is creating the most havoc,
threatening almost 80 per cent of the global banana production. The disease is so deadly that it is
sometimes referred to as ‘banana cancer’.
• Statement 2 is correct: The fungus resides below ground and infects the plant through its roots. The
infection then stops water and essential nutrients from being transported to the rest of the plant. The leaves
begin to wilt, and the stem of the plant starts turning dark brownish before the plant dies. If one plant gets
it, then it is most likely that an entire plantation can be wiped out.
• Most plant diseases – around 85 percent – are caused by fungal or fungal-like organisms. However, other
serious diseases of food and feed crops are caused by viral and bacterial organisms. Some of the common
plant diseases are Powdery Mildew, Black Spot, Bacterial Canker or Blight, Shot Hole, Black Knot, Rust,
Late Blight / Early Blight., Apple Scab etc.

Q 46.B
• India’s chemical industry is extremely diversified and can be broadly classified into bulk chemicals,
speciality chemicals, agrochemicals, petrochemicals, polymers and fertilisers. Indian colorants industry
has emerged as a key player with a global market share of ~15%.
18 www.visionias.in ©Vision IAS

DEEPAK PHOTOSTAT 9310521834


• The Indian chemicals industry stood at US$ 178 billion in 2019 and is expected to reach US$ 304 billion
by 2025. India is a net importer of Chemicals and Petrochemicals.
• The gap of domestic demand and supply of Chemicals and Petrochemicals is being bridged through
imports. To attract investment in the sector, there had been a need of spread in the duty structure of raw
materials and intermediates/finished products. With this intention, the duty structure support is provided
for crucial Tariff Lines to become domestic Chemical and Petrochemical industry, globally cost-
competitive and to become Aatma Nirbhar Bharat.
• The government has started various initiatives such as
o mandating BIS-like certification for imported chemicals to prevent dumping of cheap and
substandard chemicals into the country.
o 100% FDI is allowed under the automatic route in the chemicals sector with few exceptions that
include hazardous chemicals.
o A 2034 vision for the chemicals and petrochemicals sector has been set up by the government to
explore opportunities to improve domestic production, reduce imports and attract investments in the
sector. The government plans to implement production-link incentive system with 10-20% output
incentives for the agrochemical sector; to create an end-to-end manufacturing ecosystem through the
growth of clusters.
o The Government in the budget 2021-22 provided for a reduction in import duty on feedstock
such as ‘Naphtha’ to become globally cost-competitive for the production of Chemicals and
Petrochemicals. (The most commonly used feedstock naphtha attracts customs duty of 5 per cent
which is higher than the 2-2.5 per cent customs duty on primary petrochemicals. Chemical and
Petrochemical industry has been demanding since long back for reduction in import duty on feedstock
such as ‘Naphtha’).
o Hence, option (b) is the correct answer.

Q 47.D
• External debt (or foreign debt) is the total debt which the residents of a country owe to foreign creditors.
The debtors can be the government, corporations or citizens of that country. The debt includes money
owed to private commercial banks, foreign governments, or international financial institutions such as the
International Monetary Fund (IMF) and World Bank.
• Gross External Debt, is defined as the outstanding amount of those actual current liabilities, that require
payment(s) of principal and/or interest by the debtor, in the future as per the terms laid out in the contract
between the debtor and the creditor and that are owed to non-residents by the residents of the economy.
• In India, (Gross) External Debt is classified primarily into the following heads:
o Long and Short Term Debt: Long Term Debt is defined as debt with an ‘Original Maturity’ of more
than one year while Short Term Debt is defined as debt repayments on-demand or with an ‘Original
Maturity’ of one year or less.
ü Long-Term debt is further classified into (a) Multilateral Debt (b) Bilateral Debt (c) Export Credit
(d) (External) Commercial Borrowings (e) NRI Deposits and (d) Rupee Debt.
ü Short Term Debt is classified into (a) Trade Credits (of up to 6 months and above 6 months and
up to 1 year) (b) Foreign Institutional Investors’ (FII) Investment in Government Treasury-
Bills and Corporate Securities (c) Investment in Treasury-bills by foreign Central Banks and
International Institutions etc. and (iv) External Debt liabilities of the Central Bank and
Commercial Banks.
o Sovereign (Government) and Non-Sovereign Debt;
ü Sovereign Debt includes (a) External Debt outstanding on account of loans received by the
Government of India (GoI) under the ‘External Assistance’ programme and the civilian
component of Rupee Debt (b) Other Government debt comprising borrowings from the IMF,
defence debt component of Rupee Debt and foreign currency defence debt and (c) FII investment
in Government Securities. All remaining components of External Debt get categorized as Non-
Sovereign External Debt.
• Masala Bonds are rupee-denominated bonds issued outside India by Indian entities. They are debt
instruments which help to raise money in local currency from foreign investors. Both the government
and private entities can issue these bonds.
• Hence, option (d) is the correct answer.

Q 48.C
• Committee on Private Members’ Bills and Resolutions: This committee classifies bills and allocates time
for the discussion on bills and resolutions introduced by private members (other than ministers). This is a
19 www.visionias.in ©Vision IAS

DEEPAK PHOTOSTAT 9310521834


special committee of the Lok Sabha and consists of 15 members, including the Deputy Speaker as its
chairman. Hence statements 1 and 2 are correct.
• The Rajya Sabha does not have any such committee. The same function in the Rajya Sabha is performed
by the Business Advisory Committee of that House.

Q 49.C
• The Brus is the Indian Chinese community and is the most populous tribe in Tripura. They are spread
across Tripura, Mizoram and parts of southern Assam. Also known as Reangs in the state, they are
ethnically different from the Mizos, with their own distinct language and dialect and form one of the 21
scheduled tribes of Tripura.
• In Tripura, these tribals are recognized as Particularly vulnerable Tribal Group.
• Parts of Tripura are witnessing violent protests recently due to the proposed resettlement of Bru tribals. In
Mizoram, it is targeted by ethnic organisations demanding its exclusion from electoral rolls, forcing the
tribe to flee to Tripura. In 1997, around 37000 Bru tribals flight from Mizoram to Tripura due to ethnic
clashes.
• In a bid to ensure their permanent settlement in Tripura, an agreement was signed between the Centre and
the two states this year. According to the agreement, 32000 people residing in the camps are to
permanently settle in the state of Tripura. This agreement has led to the protests from Mizo groups and the
Bengalis in the state of Tripura. According to them settling of these migrants permanently in the state will
lead to demographic imbalance, law and order problems and will exert pressure on local resources.
• They are primarily an agrarian tribe. They mainly practiced the Jhum system of cultivation.
• The Hojagiri folk dance of Reang sub-tribe is rather well known all over the world. 'Buisu', is the most
popular festival of Reang tribes.
• Hence option (c) is the correct answer.

Q 50.A
• Recent Context – Several states have invoked Art 131 to challenge central statute. The State of Kerala
challenged the Citizens Amendment Act (CAA) by invoking the Original jurisdiction mentioned under
Article 131 of the Constitution. Similarly, The State of Chhattisgarh filed a suit under Art 131 against
NIA Act.
• The Supreme Court has three kinds of jurisdictions: original, appellate and advisory. Under its advisory
jurisdiction, the President has the power to seek an opinion from the apex court under Article 143 of the
Constitution. Under its appellate jurisdiction, the Supreme Court hears appeals from lower courts. In its
extraordinary original jurisdiction, the Supreme Court has exclusive power to adjudicate upon disputes
involving elections of the President and the Vice President, those that involve states and the Centre, and
cases involving the violation of fundamental rights.
• Article 131 provides Supreme Court exclusive original jurisdictions in federal dispute involving:
o between the Government of India and one or more states; or
o between the Government of India and any State or States on one side and one or more other
States on the other; or
o between two or more States,
• Exclusive jurisdictions means that no other court can decide such federal disputes. Original jurisdiction
means that the power to hear such disputes in the first instance, not by way of appeal. Hence statement 1
is correct.
• For a dispute to qualify as a dispute under Article 131, it has to necessarily be a) between states and b)
between the states and the Centre, and must involve a question of law or fact on which the existence of
a legal right of the state or the Centre depends. In a 1978 judgment, State of Karnataka v Union of
India, Justice P N Bhagwati had said that for the Supreme Court to accept a suit under Article 131, the
state need not show that its legal right is violated, but only that the dispute involves a legal question.
• The Supreme Court held in 2011 that when Central laws can be challenged in the state high courts
as well and also before the apex court under Article 32, normally, no recourse can be permitted to
challenge the validity of a Central law under the exclusive original jurisdiction of the Supreme
Court under Article 131. Therefore disputes involving the constitutionality of a central law are
usually questioned under article 32 in the Supreme Court and under Article 226 in the High Courts.
Hence statement 2 is not correct.
• High Courts have the jurisdiction to rule over question of law or deciding constitutionality of a
central law. The Forty-second Amendment Act, 1976 had inserted Article 226A in the Constitution:
constitutional validity of central laws not to be considered in proceedings under Article 226. The same
was repealed by the Forty-third Amendment Act, 1977. Hence statement 3 is not correct.
20 www.visionias.in ©Vision IAS

DEEPAK PHOTOSTAT 9310521834


Q 51.D
• The Ministry of Home Affairs (MHA) has constituted a national-level committee under the chairmanship
of Ranbir Singh for reform in criminal law.
• The committee, which has several leading legal academicians on board, would be gathering opinions
online, consulting with experts and collating material for their report to the government.
• The consultation exercise would start on July 4 and go on for the next three months.
• The committee has recently invited experts in the field of criminal law to participate in the exercise
through an online consultation mechanism. Questionnaires have been posted online on the possible
reforms in the Indian Penal Code, Code of Criminal Procedure and Indian Evidence Act.
• The committee includes Ranbir Singh, chairperson, Vice-Chancellor, National Law University (NLU),
Delhi; G.S. Bajpai, member and convenor, Registrar, NLU Delhi; Balraj Chauhan, member, Vice-
Chancellor, Dharmashastra NLU Jabalpur; Mahesh Jethmalani, member, Senior Advocate, Supreme Court
of India; and G.P. Thareja, member, former District and Sessions Judge, Delhi.

Q 52.A
• The Bureau of Indian Standards (BIS) is the national Standards Body of India working under the
aegis of the Ministry of Consumer Affairs, Food & Public Distribution, Government of India. Hence
statement 2 is not correct.
• It is established by the Bureau of Indian Standards Act, 1986. The organization was formerly known as
the Indian Standards Institution (ISI). The BIS Act, 1986 has now been revised as BIS Act, 2016 and
establishes BIS as the National Standards Body.
• The Bureau is a Body Corporate consisting of 25 members representing both Central and State
governments, Members of Parliament, industry, scientific and research institutions, consumer
organizations, and professional bodies; with the Union Minister of Consumer Affairs, Food and Public
Distribution as its President and with Minister of State for Consumer Affairs, Food and Public
Distribution as its Vice-President. Hence statement 1 is correct.
• The BIS Act 2016, Rules and Regulations framed thereunder authorizes BIS to undertake conformity
assessment of products, services, systems, and processes. Under the product certification scheme – I, BIS
grants a license to use the standard mark or grants certificate of conformity as per conformity assessment
schemes given in BIS (Conformity Assessment) Regulations, 2018.
• The Product Certification Schemes of BIS aims at providing Third Party assurance of quality, safety, and
reliability of products to the customer. Although the scheme itself is voluntary in nature, the Government
of India, on considerations of public health and safety, security, infrastructure requirements, and mass
consumption has enforced compulsory certification on various products through Orders issued from time
to time under various Acts. Products such as packaged mineral water, milk powder, Portland cement,
cereal-based foods, stoves, etc. are under a compulsory certification scheme.

Q 53.D
• Part III of the constitution deals with Fundamental Rights. There are six fundamental rights guaranteed by
the constitution
• Salient Features of Fundamental Rights
o Some of them are available only to the citizens while others are available to all persons whether
citizens, foreigners or legal persons like corporations or companies.
o They are not absolute but qualified.
o Some of them are negative in character, while others are positive in nature
o They are justiciable, allowing persons to move the courts for their enforcement, if and when they are
violated. They help the courts in examining and determining the constitutional validity of a law. Any
law which is in violation of the Fundamental Rights can be held void by the Supreme Court.
o They are defended and guaranteed by the Supreme Court
o They can be amended by the Constitutional Amendment (without affecting the basic structure of the
constitution)
• The Directive Principle of State Policy, Fundamental Duties and Preamble are non justiciable in nature yet
they are used by the courts in India to determine the constitutional validity of a law in India. Hence
option (d) is the correct answer.

21 ©Vision IAS

DEEPAK PHOTOSTAT 9310521834


Q 54.C
• The secret society Anushilan was initially established by Promotha Mitter in Calcutta which further
spread to other parts with different names. The Calcutta branch of the Anushilan came to be known as
Jugantar, the revolutionaries were divided into groups after the Alipur trials where Bagha Jatin escaped
the arrest and took the leadership of Jugantar. Hence statement 1 is correct.
• Jugantar and Dacca Anushilan branch broke associations from each other due to disagreements with
Aurobindo's approach of slowly building a mass base for revolution. The Dacca group instead sought
immediate action and results through political terrorism while the goal of Jugantar remained Swaraj only.
The Dacca Anushilan which was responsible for the Barrah dacoity was led by Pulin Das. Hence
statement 2 is correct.
• The Anushilan Samiti over-emphasized religion as a subject in the organization which kept the Muslims
aloof while it encouraged extremely idealistic heroism. In 1905, the samiti published “Bhavani Madir”
that incorporated a detailed plan of establishing a religious sanctuary in a secluded spot as a center of
revolutionary activities.

Q 55.D
• Permafrost is ground that remains completely frozen at zero degrees Celsius or below for at least two
years. It is made of a combination of soil, rocks, and sand that are held together by ice. The soil and ice in
permafrost stay frozen all year long.
• Permafrost is found predominantly in regions with high mountains and in Earth’s higher latitudes that are
near the North and South Poles. Almost a quarter of the land area in the Northern Hemisphere has
permafrost underneath.
• The permafrost region holds twice as much carbon as the atmosphere does (nearly 1,600 billion
tonnes). Hence statement 1 is correct.
• Permafrost contains numerous bacteria and viruses which are thousands of years old trapped in the
ice and soil. Thawing of permafrost can release these bacteria and viruses which can cause a
pandemic. Hence statement 2 is correct.
• Thawing permafrost can cause severe damage to the infrastructure and other buildings built on it
by weakening the foundation. Hence statement 3 is correct.
• Recent 20,000-tonne oil leak at an Arctic region power plant in Russia is believed to have occurred due to
permafrost thawing
• Thawing permafrost can also potentially release large amounts of carbon dioxide and methane which can
accelerate global warming.

Q 56.C
• The Sanyasi Uprisings took place in Bengal between the periods of 1770- 1820s. The Sanyasis rose in
rebellion after the great famine of 1770 in Bengal which caused acute chaos and misery. Hence
statement 1 is correct.
• The Bengal famine of 1770 led peasants whose lands were confiscated, displaced zamindars,
disbanded soldiers and poor to come together in a rebellion. They were joined by the Sanyasis (who
were originally peasants) and Fakirs. Hence statement 2 is not correct.
• However, the immediate cause of the rebellion was the restrictions imposed by the British upon
pilgrims visiting holy places among both Hindus and Muslims. Hence statement 3 is correct.
• Two famous Hindu leaders who supported them were Bhawani Pathak and a woman, Devi
Choudhurani. They attacked English factories and seized their goods, cash, arms and ammunition.
Maznoom Shah was one of their prominent leaders. It was only after a prolonged action that Warren
Hastings could subdue the sanyasis. Equal participation of Hindus and Muslims characterised the
uprisings.
• Anandamath, a semi-historical novel by Bankim Chandra Chattopadhyay, is based on the Sanyasi
Revolt. Bankim Chandra also wrote a novel, Devi Chaudhurani, as he saw the importance of women too
taking up the struggle against an alien rule that posed a threat to traditional Indian values. Hence
statement 4 is correct.

Q 57.C
• The period between 1500 B.C and 600 B.C can be divided into the Early Vedic Period or Rig Vedic
Period (1500 B.C -1000 B.C) and the Later Vedic Period (1000B.C - 600 B.C).
• Comparison between Rigvedic and Later Vedic period:
• Literature: Rigveda Samhita was the earliest text that relates to the Rigvedic period. The principal
literary sources from the Later Vedic Period are the Sama- the Yajur-, and the Atharvaveda (mainly
22 www.visionias.in ©Vision IAS

DEEPAK PHOTOSTAT 9310521834


ritual texts), the Brahmanas (manuals on ritual), and the Upanishads (Upanisads) and Aranyakas
(collections of philosophical and metaphysical discourses).
• Economy:
o The early Vedic Aryans were pastoralists. Cattle rearing was their main occupation. They reared
cattle, sheep, goats, and horses for purposes of milk, meat, and hides. The evidence for agriculture in
comparison with pastoral activities in the early portions is meagre.
o During the later Vedic phase, agriculture became the mainstay of the Vedic people. Many rituals
were introduced to initiate the process of agriculture. It also speaks of plough- ing with yokes of six
and eight oxen. Hence statement 1 is correct.
• Society:
o The family was patriarchal in nature. Monogamy was the usual norm of marriage but the chiefs at
times practiced polygamy. The Rigvedic society was largely an egalitarian society. There was no
caste division. The occupation was not based on birth.
o In the Later Vedic period, there was a rise and growth of social differentiation in the form of the
varna system. The four varnas in which society came to be divided were the Brahmanas, Kshatriyas,
vaishyas and Shudras. The growing number of sacrifices and rituals during the period made the
Brahmanas very powerful. Hence statement 2 is not correct.
• Polity:
o In the Rigvedic period, the basic unit of political organization was kula or family. Several families
joined together on the basis of their kinship to form a village or grama. The leader of grama was
known as gramani. A group of villages constituted a larger unit called visu.
o Many jana or tribes were amalgamated to form janapadas or rashtras in the later Vedic
period. The power of the king increased along with the increase in the size of the kingdom. The king
performed various rituals and sacrifices to strengthen his position. They include Rajasuya
(consecration ceremony), Asvamedha (horse sacrifice) and Vajpeya (chariot race).
• Religion:
o Gods of the Early Vedic period like Indra and Agni lost their importance in Later Vedic
period. Prajapathi (the creator), Vishnu (the protector) and Rudra (the destroyer) became prominent
during the Later Vedic period. Hence statement 3 is correct.

Q 58.A
• By exercising the powers conferred by sections 6, 25 of the Environment (Protection) Act, 1986 central
government has notified the rules for Management of Construction and Demolition Waste, 2016.
• These rules shall apply to every waste resulting from construction, re-modelling, repair, and demolition
of any civil structure of individual or organization or authority who generates construction and demolition
waste such as building materials, debris, rubble. Hence, statement 1 is not correct.
• Under these rules “waste generator” means any person or association of persons or institution, residential
and commercial establishments including Indian Railways, Airport, Port and Harbour and Defence
establishments who undertakes construction of or demolition of any civil structure which generate
construction and demolition waste. There is no exception granted to defence establishments for
generating construction and demolition waste in the construction of a civil structure. Hence
statement 2 is not correct.
• The Central Pollution Control Board shall prepare operational guidelines related to environmental
management of construction and demolition waste management and analyze and collate the data
received from the State Pollution Control Boards or Pollution Control Committee to review these rules
from time to time. Hence statement 3 is correct.
• Other important provisions under these rules are:
o Waste generators who generate more than 20 tons or more in one day or 300 tons per project in a
month shall segregate the waste into four streams such as concrete, soil, steel, wood and plastics,
bricks and mortar and shall submit a waste management plan and get appropriate approvals from the
local authority before starting construction or demolition or remodelling work
o The local authority shall issue detailed directions with regard to proper management of construction
and demolition waste within its jurisdiction in accordance with the provisions of these rules and the
local authority shall seek a detailed plan or undertaking as applicable, from a generator of
construction and demolition waste.
o State Pollution Control Board or Pollution Control Committee shall monitor the implementation of
these rules by the concerned local bodies and the competent authorities and the annual report shall be
sent to the Central Pollution Control Board and the State Government

23 www.visionias.in ©Vision IAS

DEEPAK PHOTOSTAT 9310521834


o Bureau of Indian Standards has been given the responsibility for the preparation of code of practices
and standards for use of recycled materials and products of construction and demolition waste in
respect of construction activities.
o Indian Road Congress shall be responsible for the preparation of a code of practices and standards for
the waste pertaining to the construction of roads.
o The concerned department in the State Government dealing with the land shall be responsible for
providing suitable sites for setting up of the storage, processing, and recycling facilities for
construction and demolition and hand over the sites to the concerned local authority for development,
operation, and maintenance, which shall ultimately be given to the operators by Competent Authority
and wherever above Authority is not available, shall lie with the concerned local authority.

Q 59.C
• Stem cells are cells specialized cell, capable of renewing themselves through cell division and can
differentiate into multi-lineage cells. These cells are categorized as embryonic stem cells (ESCs),
induced pluripotent stem cells (iPSCs) and adult stem cells.
• Mesenchymal stem cells (MSCs) are adult stem cells which can be isolated from human and animal
sources. Human MSCs (hMSCs) are the non-haematopoietic, multipotent stem cells with the capacity to
differentiate into mesodermal lineage such as osteocytes, adipocytes and chondrocytes as well ectodermal
(neurocytes) and endodermal lineages (hepatocytes). Mesenchymal stem cells (MSCs) are an example of
tissue or 'adult' stem cells. They are ‘multipotent’, meaning they can produce more than one type of
specialized cell of the body, but not all types. MSCs make the different specialized cells found in the
skeletal tissues. For example, they can differentiate − or specialize − into cartilage cells (chondrocytes),
bone cells (osteoblasts) and fat cells (adipocytes). These specialized cells each have their own
characteristic shapes, structures and functions, and each belongs in a particular tissue. Hence, statement
1 is correct.
• Mesenchymal stem cells (MSCs) can make several types of cells belonging to our skeletal tissues, such
as cartilage, bone and fat. Scientists are investigating how MSCs might be used to treat bone and cartilage
diseases. Some MSC research is also exploring therapies for other diseases, but the scientific basis for
these applications has not yet been established or widely accepted. Hence, statement 2 is correct.
• Some early research suggested that MSCs might also differentiate into many different types of cells that
do not belong to the skeletal tissues, such as nerve cells, heart muscle cells, liver cells and endothelial
cells, which form the inner layer of blood vessels. In some cases, it appears that the MSCs might have
fused together with existing specialized cells, leading to false conclusions about the ability of MSCs to
produce certain cell types. In other cases, the results were an artificial effect caused by chemicals used to
grow the cells in the lab.

Q 60.D
• Recently the state of Haryana enacted an Act providing 75 percent reservation in the private sector to job
seekers who have a state domicile certificate. The act states that the quota applies to all the Companies,
Societies, Trusts, Limited Liability Partnership firms, Partnership Firm and any person employing ten or
more persons and an entity as may be notified by the Government of Haryana.
• An ordinance for the purpose was approved by Haryana Cabinet in July 2020 was reserved by the
governor for the consideration of the President and ultimately shelved by the government in October
2020. However, the bill was again moved, and this time it was assented to by the governor without
reserving it for the consideration of the President. Hence, statement 1 is not correct.

24 www.visionias.in ©Vision IAS

DEEPAK PHOTOSTAT 9310521834


• Many other state governments, however, have been mulling reservations for locals in the private sector
jobs. The list includes Gujarat, Karnataka, Andhra Pradesh, etc.
• Regarding its legality, various experts say that the bill can face constitutional hurdles.
• Though the Supreme Court has capped the reservation to 50 percent in the public sector, the clause
providing for preference in jobs to the local candidates domiciled in Haryana can be seen as contravening
Article 14 of the Constitution that guarantees equality before the law and Article 19 (1)(g) which provides
for the protection of certain rights to practice any profession, or to carry on any occupation, trade or
business. Hence, statement 2 is not correct.

Q 61.A
• Many animals undertake migration for various reasons like food, reproduction, to escape threats, etc. On
the basis of migration
• Fish migration is a well-known phenomenon occurring in many fish species for different purposes in their
life cycle. Migration may occur for short distances in a river or can involve thousands of kilometres
between freshwater and marine environment.
• Based on migration behaviour, fish can be divided into Potamodromous and Diadromous fish
o Potamodromous fish are born in upstream freshwater habitats, then migrate downstream (still
in freshwater) as juveniles to grow into adults before migrating back upstream to reproduce.
(Live their whole lives in freshwater)
• Further Diadromous can be divided into
o Anadromous
o Catadromous
o Amphidromous
• Anadromous fish are born in freshwater, then migrate to the ocean as juveniles where they grow
into adults before migrating back into freshwater to reproduce. Indian shad (Hilsa ilisha) is an
example of an anadromous fish. Salmon is another well-known example of Anadromous fish.
• Catadromous fish are born in saltwater, then migrate into freshwater as juveniles where they grow
into adults before migrating back into the ocean to reproduce. Freshwater eel (Anguilla spp.) is an
example of catadromous fish.
• Hence option (a) is the correct answer.
• Amphidromous fish are born in freshwater/estuaries, then drift into the ocean as larvae before migrating
back into freshwater to grow into adults and spawn.

Q 62.D
• South Asia Co-operative Environment Programme (SACEP) is an inter-governmental organization,
established in 1982 by the governments of South Asia to promote and support protection, management,
and enhancement of the environment in the region
o The principal organs of the governance structure of SACEP are the Governing Council (GC),
Consultative Committee (CC), National Focal Points (NFP), Subject Area Focal Points, and the
SACEP Secretariat.
o Its secretariat (headquarters) is based in Colombo, Sri Lanka.
o SACEP member countries are Afghanistan, Bangladesh, Bhutan, India, Maldives, Nepal,
Pakistan, and Sri Lanka.
• South Asia Wildlife Enforcement Network (SAWEN) is an inter-governmental wildlife law
enforcement support body of South Asia.
o It promotes regional cooperation to combat wildlife crime in South Asia. It focuses on policy
harmonization; institutional capacity strengthening through knowledge and intelligence sharing, and
collaboration with regional and international partners to enhance wildlife law enforcement in the
member countries.
o The operational modality of SAWEN consists of a General Assembly as an apex body of SAWEN
comprising minister of the concerned ministry or his/her senior-level government delegate, principal
focal person of the SAWEN member countries, and one representative from each of the associate
members and international/partner members.
o Its secretariat (headquarters) is based in Kathmandu, Nepal.
o SAWEN member countries are Afghanistan, Bangladesh, Bhutan, India, Maldives, Nepal,
Pakistan and Sri Lanka.
• Hence, option (d) is the correct answer.

25 www.visionias.in ©Vision IAS

DEEPAK PHOTOSTAT 9310521834


Q 63.C
• Nitisara, written by Kamandaka, is a text like Arthasastra. It emphasizes the importance of the royal
treasury and mentions various sources of revenue during the period of the Gupta empire. The many
ambitious military campaigns of kings like Samudragupta must have been financed through revenue
surpluses. Gupta inscriptions reveal some details about the revenue department. The akshapataladhikrita
was the keeper of royal records. Gupta inscriptions mention the terms Bhaga, Hiranya, Udianga etc. as
the taxes.
• Different types of taxes during Gupta's rule include
o Bhaga: King’s customary share of the produce normally amounting to one-sixth of the produce paid
by cultivators Bhoga Periodic supply of fruits, firewood, flowers, etc., which the village had to
provide to the king
o Udianga: Either a sort of police tax for the maintenance of police stations or a water tax. Hence, it
was also an extra tax.
o Hiranya: Literally, it means tax payable on gold coins, but in practice, it was probably the king’s
share of certain crops paid in kind.
o Vata-Bhuta: Different kinds of cess for maintenance of rites for the winds (Vata) and the spirits
(bhuta) Halivakara A plough tax paid by every cultivator owning a plough
• Hence option (c) is the correct answer.

Q 64.D
• A schedule is a part of an act that contains extra information that has not been written in the body of the
legislative document. It is an appendix attached to any act, supplementing the legislation with additional
information that is not mentioned under the main text of the articles.
• In the case of Indian constitution, Schedules are tables which contains additional details not mentioned in
the articles of constitution.
• Indian constitution originally contained eight schedules. Later four more schedules were added by
different amendments, making it a total of twelve schedule.
• The second schedule contains provisions relating to the emoluments, allowances, privileges and so on
of – a) President, b) Governor of states, c) Speaker and Deputy speaker of Lok Sabha, d) Chairman and
Deputy Chairman of Rajya Sabha, e) Speaker and Deputy Speaker of the Legislative Assembly of states,
f) Chairman and Deputy Chairman of the Legislative Council of States, g) Judges of Supreme Court, h)
Judges of High Court, i) Comptroller and Auditor General of India
• The third schedule contains forms of Oaths or Affirmations for – a) Union Ministers, b) Candidates
for election to the President, c) Members of Parliament, d) Judges of Supreme Court, e) Comptroller
and Auditor General of India f) State Ministers, g) Candidates for election to the state legislature, h)
Members of the state legislature, i) Judges of High Court
• Hence option (d) is the correct answer.

Q 65.D
• Commencement of a Sitting of the House of Parliament is duly constituted when it is presided over by the
Speaker or any other member competent to preside over a sitting of the House under the Constitution or
the Rules of the House. It is, therefore, necessary that the Speaker, the Deputy Speaker or a member of the
Panel of Chairmen presides over the House at the hour fixed for the commencement of a sitting and also
so long as the sitting lasts. If no such person is present, such other person as may be determined by the
House can act as Speaker. Hence statement 1 is not correct.
• Presence of Ministers is not necessary for constituting a sitting of the House. Hence statement 2 is not
correct.
• When members of the two Houses of Parliament assemble together to hear the President's Address, it does
not constitute a sitting of the Rajya Sabha since a sitting of the Rajya Sabha is duly constituted when it is
presided over by the Chairman or a member competent to preside over a sitting of the Rajya Sabha under
the Constitution or the Rules of Procedure and Conduct of Business in the Rajya Sabha. Hence statement
3 is not correct.

Q 66.B
• Recent Context: On December 12, 2020, the Paris Climate Agreement completed its fifth anniversary.
Paris Agreement aims to limit the increase in global warming temperatures well below two degrees
Celsius, a target the world is yet to achieve. To commemorate the occasion, the United Kingdom had
called for a Global Climate Summit. It is also called International Climate Ambition Summit.

26 www.visionias.in ©Vision IAS

DEEPAK PHOTOSTAT 9310521834


• The International Climate Action Summit was organized and co-hosted by United Nations, United
Kingdom and France, in partnership with Chile and Italy. Hence, statement 1 is not correct.
• The summit aimed to highlight the achievements of countries, cities, businesses towards net-zero carbon
emission and enhancing climate finance. Hence, statement 2 is correct.
• The summit is a predecessor to 26th COP scheduled to be held at Glasgow in 2021. The member countries
presented a detailed deliberation of NDCs, long-term strategies to achieve zero-emission and adaptation
plans and policies.
• The summit featured United Kingdom, India, Canada, Japan, China, South Korea and European Union.
Australia, the supporter of fossil fuels has been kept out of the summit. The other notable absences were
Brazil, Mexico, Saudi Arabia, Poland and Turkey. The United States did not attend the summit as
President Trump walked out of the Paris Agreement.
• The United Nations Environment Programme recently released the Emissions Gap Report, 2020. The
report provided the review of the difference between where the Greenhouse gas emissions are predicted to
be in 2030 and where they should be to prevent catastrophic climate change events.
• According to the reports, US has 25% of all emissions, Europe has 22% of emissions, China has 13% and
India has only 3% of the emissions of the world.

Q 67.D
• Indo-China bilateral trade developed slowly and steadily in the formative years of 1950s. During 1990s,
Indo-China bilateral trade began to expand rapidly, particularly after mid-1990s.
• In 2020, China reclaimed its position at the top of the list of India’s major trade partners, replacing the
United States that had climbed to number 1 in 2019.
• Trade between India and China from January to December 2020 stood at $77.67 billion. Though lower
than the $85.47 billion traded between the countries in the 2019 calendar year, this figure was still higher
than the $75.95 billion traded between India and the US last year.
• While India has been trying to reduce its trade imbalance and dependence on Chinese imports for several
years now, it was only in 2018 that the US surpassed the value of goods that China traded with India in a
financial year.
• Trends in China trade
o Electrical machinery and equipment, at $17.82 billion, and nuclear reactors, boilers, machinery, and
mechanical appliances, at $12.35 billion, continued to top the list of goods imported from China in
2020 — even though the imports of these goods dropped by nearly 11 per cent in the last
calendar year compared to one year previously.
o Exports of Indian iron and steel to China jumped by a massive 319.14 per cent over 2019, with
shipments touching $2.38 billion during January to December 2020. Iron and steel exports to
China in 2019 were around $567 million. Export of ores, slag, and ash increased by 62 per cent to
$3.48 billion in 2020 from $2.15 billion in 2019.
o Overall, exports to China in 2020 were $17.12 billion — around 10.70 per cent higher than in
2019.
o Hence, option (d) is correct.

Q 68.D
• The Competition Act, 2002, as amended by the Competition (Amendment) Act, 2007, follows the
philosophy of modern competition laws. The Act prohibits anti-competitive agreements, abuse of
dominant position by enterprises and regulates combinations (acquisition, acquiring of control and M&A),
which causes or likely to cause an appreciable adverse effect on competition within India.
• The objectives of the Act are sought to be achieved through the Competition Commission of India, which
has been established by the Central Government with effect from 2003. CCI consists of a Chairperson and
6 Members appointed by the Central Government.
• Competition Commission of India aims to establish a robust competitive environment through:
o Proactive engagement with all stakeholders, including consumers, industry, government and
international jurisdictions.
o Being a knowledge-intensive organization with high competence level.
o Professionalism, transparency, resolve and wisdom in enforcement.
• It is the duty of the Commission to eliminate practices having adverse effect on competition,
promote and sustain competition, protect the interests of consumers and ensure freedom of trade in
the markets of India.

27 www.visionias.in ©Vision IAS

DEEPAK PHOTOSTAT 9310521834


• The Commission is also required to give opinion on competition issues on a reference received from a
statutory authority established under any law and to undertake competition advocacy, create public
awareness and impart training on competition issues.
• The CCI can take up the investigation suo moto to check if companies have plotted to increase
prices of several products. The Competition Commission of India (CCI) has launched an investigation
against steel companies for allegedly forming a cartel, the latest instance of the country’s anti-trust
regulator scrutinising businesses to check if they are colluding to fix rates.
• The Competition Commission of India (CCI) can impose fines on companies/individuals indulging
in anti-competitive practices. CCI has slapped a total penalty of Rs 4 lakh on the Federation of
Publishers and Booksellers Association in India (FPBAI) and two individuals for indulging in anti-
competitive practices.
• Hence, all the statements are correct.

Q 69.C
• The Pallavas established their kingdom in Tondaimandalam with its capital at Kanchipuram. Their
rule continued till Tondaimandalam was captured and annexed by the Imperial Cholas in the beginning of
the tenth century A.D.
• The Pallavas introduced the art of excavating temples from the rock. It was a gradual evolution starting
from the cave temples to monolithic rathas and culminated in structural temples. The development of
temple architecture under the Pallavas can be seen in four stages.
o Mahendravarman I introduced the rock-cut temples. This style of Pallava temples are seen at
places like Mandagappattu, Mahendravadi, Mamandur, Tiruchirappalli etc.
o The second stage of Pallava architecture is represented by the monolithic rathas and Mandapas
found at Mamallapuram. Narasimhavarman I took the credit for these wonderful architectural
monuments. The five rathas, popularly called as the Panchapanadava rathas, signifies five different
styles of temple architecture.
o In the next stage, Rajasimha introduced the structural temples. These temples were built by using the
soft sand rocks. The Kailasanatha temple at Kanchi and the Shore temple at Mamallapuram
remain the finest examples of the early structural temples of the Pallavas.
o The last stage of the Pallava art is also represented by structural temples built by the later Pallavas.
The Vaikundaperumal temple, Muktheeswara temple and Matagenswara temples at Kanchipuram
belong to this stage of architecture.
• Virupaksha temple is built on the model of the Kailasanatha temple at Kanchipuram. It was built by one
of the queens of Vikramaditya II who belonged to Chalukya dynasty.
• Hence option (c) is the correct answer.

Q 70.D
• Grafting: It is the process of inserting a part of one plant in to another in such a way that a union will take
place and combination will continue to grow as a plant. The part of the graft which forms the upper
portion is called as root stock or under stock. This is most suitable for repairing the damage to trees in a
better way. Rootstock is the plant above which scion is inserted. E.g: Mango, citrus, plum. Hence option
(d) is the correct answer.
• Budding is a type of asexual reproduction in which a new organism develops from an outgrowth or bud
due to cell division at one particular site. The small bulb-like projection coming out from the yeast cell is
called a bud. Since the reproduction is asexual, the newly created organism is a clone and excepting
mutations is genetically identical to the parent organism. Organisms such as hydra use regenerative cells
for reproduction in the process of budding.
• Layering is a means of plant propagation in which a portion of an above-ground stem grows roots while
still attached to the parent plant and then detaches as an independent plant. Layering has evolved as a
common means of vegetative propagation of numerous species in natural environments. E.g.-Clematis,
black berry, raspberry, Indian rubber, guava.
• The term cutting refers to any part of a plant that has been severed from the mother plant for purposes of
propagation. Four common types of cuttings are stem cuttings, leaf cuttings, leaf-bud cuttings, and root
cuttings. To be successful as an asexual propagule, a cutting must develop adventitiously the organs it
lacks. A stem must develop roots, a leaf cutting must develop roots and shoots, a leaf-bud cutting must
develop roots, and a root cutting must develop shoots. E.g: Tea, rhododendrum, camellia, Jasmine etc.

28 www.visionias.in ©Vision IAS

DEEPAK PHOTOSTAT 9310521834


Q 71.A
• The International Investment Position (IIP) is a statistical statement that shows at a point in time the value
of financial assets of residents of an economy that are claims on nonresidents or are gold bullion held as
reserve assets; and the liabilities of residents of an economy to non-residents. Hence, statement 1 is
correct.
• The difference between the assets and liabilities is the net position in the IIP and represents either a net
claim on or a net liability to the rest of the world.
• The IIP represents a subset of the assets and liabilities included in the national balance sheet. In addition
to the IIP, the national balance sheet incorporates non-financial assets as well as financial assets and
liability positions between residents
• The IIP of a country is a financial statement that provides the value and composition of its external
financial assets. A positive IIP says that the nation is the creditor and a negative IIP says that it is
debtor. Hence, statement 2 is not correct.
• The External Foreign Assets determine the value of a nation’s currency greatly. When the currency
appreciates, the value of foreign assets decreases. When the currency depreciates, the value of foreign
assets increases.
• Therefore, if a country is net debtor, currency depreciation will raise its burden of a foreign currency debt.
• On June 30, 2020, the Reserve Bank of India released its data on International Investment Position, IIP.
According to the data, the direct investment in the country rose by 19 billion USD and the portfolio
investment declined by 13.7 billion USD. Also, the international financial assets of Indian residents have
increased by 73.9 billion USD.

Q 72.D
• Nangiarkoothu, the performance by the Nangiars, is a derivative of Koodiyattam, the ancient Sanskrit
theatre.
• This ritualistic art form is believed to have its roots in the nirvahanam of Kalpalathika, Subhadra’s maid
in the second act of the play Subhadradhanajayam of Kulasekhara Alwar.
• In Nangiarkoothu, all the characters in a story are enacted by a woman.
• The text for Nangiarkoothu is Sreekrishnacharitham comprising 217 slokas taken from various scriptures
such as the Bhagavatham, Geethagovindam, Harivamsapuranam, Brahmandapuranam, Kamsavaho, etc.
• It starts with a description of Mathura and then goes on to narrate stories of Lord Krishna’s
childhood pranks, episodes in the life of Lord Krishna and culminates with the abduction of
Subadra, thereby linking it with the play Subadradhananjayam.
• Nangiarkoothu used to be staged at Koothambalams, theatres with exquisite carvings and paintings that
were part of famous temples in Kerala.
• Over the years this form has evolved by pushing its boundaries in terms of performance space as well as
acting texts and has slowly transformed into an aesthetic art form rather than a mere ritualistic
performance.
• The uniqueness of this form is that even though this is performed by a single actor assuming the roles of
different characters of the story, the art form encompasses both dance and drama.

Q 73.D
• Peatland:
o Peatlands are a type of wetlands that are among the most valuable ecosystems on Earth and can be
protected and classified under the Ramsar Convention on Wetlands. Hence, statement 3 is correct.
o The term ‘peatland’ refers to the peat soil and the wetland habitat growing on its surface.
o In these areas, year-round waterlogged conditions slow the process of plant decomposition to such an
extent that dead plants accumulate to form peat, a spongy, organic material formed by partially
decayed wetland plants.
o Over millennia this material builds up and becomes several metres thick. Peatlands contain 30 per
cent of the world’s soil carbon.
o They occur in almost every country on Earth and in every climatic zone, currently covering 3% of the
global land surface. Hence statements 2 and 4 are correct.
o Peatlands are the largest natural terrestrial carbon store; the area covered by near-natural peatland
worldwide (>3 million km2) sequesters 0.37 gigatonnes of carbon dioxide (CO2) a year – storing
more carbon than all other vegetation types in the world combined. Hence, statement 1 is correct.
o Their degradation due to drainage, fire, agricultural use and forestry can trigger the release of the
stored carbon in a few decades.
o Draining peatlands reduces the quality of drinking water due to pollution from dissolved compounds.
29 www.visionias.in ©Vision IAS

DEEPAK PHOTOSTAT 9310521834


• Peatlands can be categorized into three groups:
o Bogs receive very little surface water flow and are among the most acidic peatlands. They are
dominated by shrubs.
o Fens are peatlands associated with moving water, either along a river or lake, or with a stream that
flows into or out of the peatland. Fens range from very acidic (where the plants resemble those found
in bogs) to mildly acidic, and are dominated by a combination of sedges and shrubs.
o Peat swamps are peatlands dominated by trees. There are many different types of these forested
wetlands.

Q 74.A
• Article 14 of Indian constitutions deals with the Equality before law and equal protection of laws. It
says that the State shall not deny to any person equality before the law or the equal protection of the laws
within the territory of India.
• The first concept denotes
o the absence of any special privileges in favour of any person
o the equal subjection of all persons to the ordinary law of the land administered by ordinary law courts,
and
o no person (whether rich or poor, high or low, official or non-official) is above the law. Hence
option 1 and 2 are correct.
• While, the second concept explains that
o the equality of treatment under equal circumstances, both in the privileges conferred and liabilities
imposed by the laws
o the similar application of the same laws to all persons who are similarly situated
o the like should be treated alike without any discrimination. Hence option 3 is not correct.
o Thus, the former is a negative concept while the latter is a positive concept. However, both of them
aim at establishing equality of legal status, opportunity and justice.

Q 75.A
• Alluvial soil:.
o They are widespread in the northern plains and the river valleys.
o These soils cover about 40 per cent of the total area of the country.
o Through a narrow corridor in Rajasthan, they extend into the plains of Gujarat.
o In the Peninsular region, they are found in deltas of the east coast and in the river valleys.
o The alluvial soil consists of various proportions of sand, silt and clay.
o In the upper reaches of the river valley i.e. near the place of the break of slope, the soils are coarse.
Such soils are more common in piedmont plains such as Duars, Chos and Terai. Hence, pair 1 is
correctly matched.
o They are generally rich in potash but poor in phosphorous.
o In the Upper and Middle Ganga plain-
ü Khadar is the new alluvium and is deposited by floods annually, which enriches the soil by
depositing fine silts.
ü Bhangar represents a system of older alluvium, deposited away from the flood plains. Both the
Khadar and Bhangar soils contain calcareous concretions (Kankars).
ü These soils are more loamy and clayey in the lower and middle Ganga plain and the
Brahamaputra valley.
o The sand content decreases from the west to east.
o The colour of the alluvial soils varies from the light grey to ash grey. Its shades depend on the depth
of the deposition, the texture of the materials, and the time taken for attaining maturity.
• Laterite soil:
o Laterite has been derived from the Latin word ‘Later’ which means brick.
o The laterite soils develop in areas with high temperature and high rainfall. These are the result of
intense leaching due to tropical rains. With rain, lime and silica are leached away, and soils rich in
iron oxide and aluminium compound are left behind. Hence, pair 2 is correctly matched.
o Humus content of the soil is removed fast by bacteria that thrives well in high temperature.
o These soils are poor in organic matter, nitrogen, phosphate and calcium, while iron oxide and potash
are in excess. Hence, laterites are not suitable for cultivation. However, application of manures and
fertilisers are required for making the soils fertile for cultivation.

30 www.visionias.in ©Vision IAS

DEEPAK PHOTOSTAT 9310521834


o They are prone to erosion and degradation due to their position on the landscape. After adopting
appropriate soil conservation techniques particularly in the hilly areas of Karnataka, Kerala and Tamil
Nadu, this soil is very useful for growing tea and coffee.
o Red laterite soils in Tamil Nadu, Andhra Pradesh and Kerala are more suitable for tree crops like
cashewnut.
o The laterite soils are commonly found in Karnataka, Kerala, Tamil Nadu, Madhya Pradesh and the
hilly areas of Odisha and Assam.
• Red soils:
o Red soil develops on crystalline igneous rocks in areas of low rainfall in the eastern and southern part
of the Deccan Plateau. Hence, pair 3 is correctly matched.
o Along the piedmont zone of the Western Ghat, long stretch of area is occupied by red loamy soil.
o Yellow and red soils are also found in parts of Odisha and Chhattisgarh and in the southern parts of
the middle Ganga plain.
o The soil develops a reddish colour due to a wide diffusion of iron in crystalline and metamorphic
rocks. It looks yellow when it occurs in a hydrated form.
o The fine-grained red and yellow soils are normally fertile, whereas coarse-grained soils found in dry
upland areas are poor in fertility. They are generally poor in nitrogen, phosphorous and humus.

Q 76.D
• Invasive Alien Species (IAS) are animals, plants, or other organisms that are introduced into places
outside their natural range, negatively impacting native biodiversity, ecosystem services, or human well-
being.
• IAS are one of the biggest causes of biodiversity loss and species extinctions and are also a global threat
to food security and livelihoods.
• IAS are compounded by climate change. Climate change facilitates the spread and establishment of many
alien species and creates new opportunities for them to become invasive.
• Some of the examples of IAS in India are
o Lantana camera
o Siam weed
o Parthenium hysterophorus
o Mexican devil
o Giant African snail
o Cotton Mealy Bug
o serpentine leafminer
o spiralling whitefly
o Water hyacinth
o Mikania micrantha
o Mesquite
• Some common characteristics of Invasive Alien Species which enable them to spread so rapidly and
extensively:
o Rapid reproduction enables them to quickly grow their numbers and out-compete the native
animals and plants for food.
o High dispersal ability enables them to spread over a wide area with ease and in a short amount
of time.
o Phenotypic plasticity is the ability of an organism to change in response to stimuli or inputs
from the environment. It enables the IAS to survive on various food types and in a wide range
of environmental conditions.
o Short dormancy period
o Gregarious in nature
• Hence option (d) is the correct answer.

Q 77.C
• Bal Gangadhar Tilak (July 1856 – August 1920) was an Indian nationalist, teacher, and independence
activist.
• He was one of the Lal Bal Pal triumvirate.
• The British colonial authorities called him "The father of the Indian unrest." He was also conferred with
the title of "Lokmanya", which means "accepted by the people (as their leader)".Mahatma Gandhi called
him "The Maker of Modern India".

31 www.visionias.in ©Vision IAS

DEEPAK PHOTOSTAT 9310521834


• Tilak was one of the first and strongest advocates of Swaraj ("self-rule") and a strong radical in Indian
consciousness. He is known for his quote in Marathi: "Swarajya is my birthright and I shall have it!".
• Tilak’s concept of Swarajya is based on the Vedanta philosophy. He pronounced the idea and invented
methods to achieve the goal. Throughout his life he was doing the Lokasangraha to achieve the goal of
Swarajya. Lokasangraha means that an enlightened person should teach people about how to do the work
as per the Chaturvarnya system with Nishkam Karmayoga i.e. doing work skillfully and successfully
without having desires attached to the final outcome. Hence option (b) is correct.
• In Tilak’s political theory equality, liberty, justice and all the natural rights find their origin in one term
called Swarajya, which, has its foundations in Vedanta philosophy. Therefore, while speaking against the
scourge of untouchability, as a true Vedanti Tilak could declare – “If God were to tolerate untouchability I
would not recognize Him as God at all.”
• Tilak and Swami Vivekananda (January 1863 – July 1902) were contemporaries. They met
accidentally while traveling in a Pune-bound train in 1892. They stayed connected during their struggle
for India's independence. Hence option (a) is correct.
• He vehemently criticized the British rule and endeavored to awaken the political consciousness of people
through two weekly newspapers that he owned and edited, “Kesari”, published in Marathi and “The
Mahratta”, published in English. Hence option (c) is not correct.
• Tilak’s campaign against the British colonial rule was anchored on reclaiming Indian heritage and culture,
some of which were reflected in his 1892 paper, “The Orion, or Researches into the Antiquity of the
Vedas”, which drew on the vast knowledge of the Hindu Vedas and Avesta, the sacred scripture of
Zoroastrianism. It was a search into the antiquity of the Aryan-Vedic culture, challenging the ancientness
of the Bible and establishing the primacy of the Vedic literature. Hence option (d) is correct.

Q 78.C
• Bihar is set to become north India's first "bird ringing station" for observation, monitoring and research on
migratory birds, the fourth state in the country after Tamil Nadu, Rajasthan and Orissa.
• Bird ringing:
o Rings are placed on the legs of birds. The rings come with chips that help in tracking the origin of the
birds and the route taken by these during migration. It is not an in-situ conservation technique but it
facilitates monitoring and record-keeping of wild avian species, especially migratory birds. Hence
option (c) is the correct answer.
o Ringing is conducted to study the biology of birds:
ü the ways and duration of their migration, longevity, mortality, population studies, such as age
structure, population size, territoriality, survival and reproduction rate, the behavioural
characteristics of individuals.
ü It also helps to address the problem of bird conservation.
Q 79.B
• Producer Price Index (PPI) measures the average change in the price of goods and services either as they
leave the place of production, called output PPI or as they enter the production process, called input PPI.
PPI estimates the change in average prices that a producer receives.
• Consumer Price Index (CPI) is a measure of the change in retail prices of goods and services consumed by
defined population group in a given area with reference to a base year. Hence, statement 1 is not correct.
• A wholesale price index (WPI) is an index that measures and tracks the changes in the price of goods in
the stages before the retail level.
• PPI is different from WPI on the following grounds:
o WPI captures the price changes at the point of bulk transactions and may include some taxes levied
and distribution costs up to the stage of wholesale transactions. PPI measures the average change in
prices received by the producer and excludes indirect taxes. Hence, statement 2 is correct.
o Weight of an item in WPI is based on net traded value whereas in PPI weights are derived from the
Supply Use Table.
o PPI removes the multiple counting bias inherent in WPI.
o WPI does not cover services and whereas PPI includes services.

Q 80.A
• The government's incubation fund DISHA seeks to promote indigenous manufacturing of
unmanned aerial vehicles (UAVs).
• DISHA stands for Drones for Infra, Security, Healthcare and Agriculture. Infrastructure, Security,
Healthcare, and Agriculture are the four broad focus areas as there are massive opportunities that could
benefit the lives of common Indians.
32 www.visionias.in ©Vision IAS

DEEPAK PHOTOSTAT 9310521834


• The DISHA fund will give financial support to Indian startups and researchers to manufacture drones or
UAVs for use in four sectors -- infrastructure, security, healthcare and agriculture.
• Currently, drones or components manufactured by China form the backbone of India’s drone industry
• The fund being conceptualized by NITI Aayog aims at bridging the gap between academia and industry
and seeks to make Indian industry atmanirbhar in drone-making capabilities.
• Due to the COVID triggered economic slowdown, it was recently allocated less than Rs 500 crore.

Q 81.C
• Udayagiri and Khandagiri Caves have derived their names, owing to their location on two hills,
Udayagiri and Khandagiri. Mentioned as Kumari & Kumara Parvatas in Hathi Gumpha inscription, the
two caves face each other across the road. These two hills represent one of the earliest groups of Jaina
rock-cut architecture in Odisha in the field of architecture, art and religion. The caves of Udayagiri and
Khandagiri, called Lena in the inscriptions, were dug out mostly during the reign of Kharavela for
the abode of Jaina monks during 1st century BC. Hence option 1 is correct.
• The Early Chalukyas chose the finely-grained and horizontally-stratified sandstone cliffs
of Badami (Bijapur District), for rock excavations, which facilitated the excavation of comparatively
large cave-temples and the execution of fine sculptures and intricate carvings in them. There are four
such cave-temples, three Brahmanical and the fourth Jaina. The earliest of them (Cave 3), dedicated
to Vishnu, is the largest of the series and was excavated, according to its inscription, in Saka 500, i.e. A.D.
578, by Mangalesa, a powerful ruler. It was followed in quick succession by the other two, Cave 2, the
smallest, also dedicated to Vishnu, and Cave 1, of medium size, dedicated to Siva. The Jaina cave-temple
at the very top of the hill is later by about a century from the rest. Hence option 2 is correct.
• The Ellora Jain Caves are five in number. These caves are separated from the other caves by a distance
of a few hundred yards. There are two main caves and two unfinished caves among the five. These
caves date back to the 9th and 10th centuries and belong to the Jain sect called Digambara. Hence
option 3 is correct.
• Ravan phadi cave is the only Brahmanical cave temple at Aihole. The rock-cut Hindu temple of
Ravana Phadi in Aihole, Karnataka, was made during 550 AD. A big colossal pillar placed on a
quadrangle base is located in front of the entrance hall to the temple. There is also figure of Nandi Bull,
ride of Lord Shiva, in a seated posture, near the entrance. On either side of the entrance there are hallways
with pillars that enhance the beauty of the architecture. More over the entrance area is ornamented with
stone carved columns. There are artworks of well built deities on the sides of the entrance, which probably
depict the sentinel of wealth, Kubera. Hence option 4 is not correct.

Q 82.B
• Manimekalai is a Tamil epic composed by Buddhist poet Sattanar of Madurai. It is an "anti-love story",
a sequel to the "love story" in the earliest Tamil epic Silappadikaram. It is a sort of Buddhist supplement
to the Silappadikaram.
• It describes the story of Manimegalai (the daughter of Kovalan and Madhavi of the earlier epic), how she
preserves her chastity from Prince Udaya Kumaran and becomes a Buddhist nun to preach its
doctrines. Besides containing a good deal of social and historical information, it is the only important
ancient work that gives glimpses of the development of the fine arts in the Sangam age. Hence option
(b) is the correct answer.

Q 83.A
• The financial sector plays an important role in a modern economy by ensuring financial intermediation,
i.e. the channelling of funds from savers to investors.
• A sound and efficient financial sector encourages the accumulation of savings and enables their allocation
to the most productive investments, thus supporting innovation and economic growth.
• The soundness of banks can be assessed through the following indicators:
o Non-performing loans to total loans (the NPL ratio) ratio relates the nominal value of non-
performing loans to all loans. The ratio shows the extent of deterioration of the quality of loans
granted by the banks. The higher the ratio, the worse the quality of the assets, and consequently
the higher the expected losses. Hence option 2 is correct.
o The capital adequacy ratio (CAR) shows the solvency of banks. It relates the value of regulatory
capital, i.e. capital instruments recognised by the banking regulation, to risk-weighted assets. It is an
indicator of banks' capacity to absorb losses. The higher the ratio, the more the banks can absorb
losses without endangering their solvency. Hence option 1 is correct.

33 www.visionias.in ©Vision IAS

DEEPAK PHOTOSTAT 9310521834


o The RoE ratio relates banks' net income (i.e. profits after-tax) to total capital. It is an indicator of
banks' overall profitability. A high profitability suggests that banks are in a favourable position to
increase their capital buffer in the immediate future, namely through retained earnings.
o The loan-to-deposit ratio (LDR) is used to assess a bank's liquidity by comparing a bank's total loans
to its total deposits for the same period. The LDR is expressed as a percentage. If the ratio is too
high, it means that the bank may not have enough liquidity to cover any unforeseen fund
requirements. Hence, the loan-to-deposit ratio (LTD ratio) is a common indicator that helps
assess whether banks have stable funding. Hence option 3 is not correct.

Q 84.A
• Recent Context: GAVI was announced as one of the organisations leading the COVAX vaccine
allocation plan, created to ensure that any new COVID-19 vaccine would be shared equally between the
world's richest and poorest countries.
• GAVI, officially Gavi, the Vaccine Alliance is a public-private global health partnership with the goal of
increasing access to immunization in poor countries. It was previously known as Global Alliance for
Vaccines and Immunization. Hence, statement 1 is correct. GAVI was created in 2000 as a successor to
the Children's Vaccine Initiative, which had been launched in 1990. Hence, statement 2 is not correct.
• GAVI brings together developing countries, donor governments, the World Health Organization,
UNICEF, the World Bank, the vaccine industry in both industrialised and developing countries, research
and technical agencies, civil society etc.
• GAVI has observer status at the World Health Assembly.
• It currently supports the immunization of almost half the world's children, giving it power to negotiate
better prices for the world's poorest countries and remove the commercial risks that manufacturers faced
in serving this market. It also provides funding to strengthen health systems and train health workers
across the developing world. To date Gavi has helped immunize over 760 million children, preventing
over 13 million deaths worldwide.
• Gavi shares the cost that implementing countries pay for vaccines, which has resulted in more than 495
vaccine introductions and campaigns, dramatically boosting immunisation against virulent diseases.

Q 85.D
• Recent Context: Recently, President’s rule was imposed in Union Territory of Puducherry.
• Recent Context – Recently, President’s rule was imposed in Union Territory of Puducherry. The union
territory's Assembly has been kept under suspended animation.
• The state legislative Assembly consists of representatives directly elected by people. Also, The governor
can nominate one member from the Anglo Indian Community, if the community is not adequately
represented in the Assembly.
• In case of UT’s, not all of them have legislature. Out of 9 UT’s, Only 3 UT’s (Delhi, Puducherry and
Jammu and Kashmir) have legislature. The Government of Union Territories Act provides for a 33-
member House for Puducherry of whom three are to be nominated by the Central government. The Union
Territory of Delhi has 70 member legislature without any nominated member. Similarly the UT of
Jammu and Kashmir has maximum strength of its Assembly as 107 seats. Under the new law, LG can
nominate two women representatives in the J&K Assembly if he/she feels there is an inadequate female
representation Thus not all UT’s with legislature have nominated member. Hence statement 1 is not
correct
• The ‘Council of States’ which is also known as Rajya Sabha, represents the states and UT’s of the Indian
Union. However, not all UT’s are allocated seats in Rajya Sabha. Only UT’s of Delhi, Puducherry and
Jammu and Kashmir have representation in Rajya Sabha. Hence statement 2 is not correct.

Q 86.D
• Alternative investment funds (AIFs) refers to any privately pooled investment fund, (whether from
Indian or foreign sources), in the form of a trust or a company or a body corporate or a Limited
Liability Partnership (LLP) which are not presently covered by any Regulation of SEBI governing
fund management (like, Regulations governing Mutual Fund or Collective Investment Scheme)nor
coming under the direct regulation of any other sectoral regulators in India-IRDA, PFRDA, RBI. Hence,
in India, AIFs are private funds which are otherwise not coming under the jurisdiction of any
regulatory agency in India.
• Thus, the definition of AIFs includes venture Capital Fund, Angel funds, Fund of funds, commodity
funds, Debt Funds, infrastructure funds, etc.

34 www.visionias.in ©Vision IAS

DEEPAK PHOTOSTAT 9310521834


o “Angel Fund” is a sub-category of Venture Capital Fund under Category I Alternative
Investment Fund that raises funds from angel investors (a high-net-worth individual who
provides financial backing for small startups or entrepreneurs). In the case of an angel fund, it
shall only raise funds by way of issue of units to angel investors.
o Debt fund is an Alternative Investment Fund (AIF) which invests primarily in debt or debt
securities of listed or unlisted investee companies according to the stated objectives of the Fund.
o Fund of Funds, in general parlance as gathered from publicly available sources s an investment
strategy of holding a portfolio of other investment funds rather than investing directly in stocks,
bonds or other securities. In the context of AIFs, a Fund of Fund is an AIF which invest in another
AIF.
• Hence all the statements are correct.

Q 87.B
• Recently the Union Cabinet approved a proposal by the Home Ministry to dissolve the Puducherry
Assembly and impose President’s rule in the Union Territory. The notification said the President had
received a report from the administrator of the UT on February 22 and “after considering the report and
other information”, the President was “satisfied that a situation has arisen in which the administration of
the Union Territory of Puducherry cannot be carried on in accordance with the provisions of the
Government of Union Territories Act, 1963 (20 of 1963).” Hence statement 2 is not correct.
• Since the formation of the Republic, President’s Rule under Article 356 has been imposed in states in over
100 occasions. Proclamation of President's rule under Article 356 is on the advice of the Council of
Ministers tendered under Article 74. Hence statement 3 is correct.
• According to the Article 356, “(1) If the President, on receipt of the report from the Governor of the
State or otherwise, is satisfied that a situation has arisen in which the government of the State
cannot be carried on in accordance with the provisions of this Constitution”.
• In Bommai case (1994), the following propositions have been laid down by the Supreme Court on
imposition of President’s Rule in a state under Article 356:
o The presidential proclamation imposing President’s Rule is subject to judicial review. Hence
statement 1 is correct.
o The satisfaction of the President must be based on relevant material. The action of the president
can be struck down by the court if it is based on ‘irrelevant’ or ‘extraneous’ grounds or if it was
found to be malafide or perverse. Note: Satisfaction has to be subjective satisfaction of the Cabinet
and not the President.
o The court cannot go into the correctness of the material or its adequacy but it can see whether it is
relevant to the action
o Burden lies on the Centre not the state government to prove that relevant material exist to
justify the imposition of the President’s Rule.
o The state legislative assembly should be dissolved only after the Parliament has approved the
presidential proclamation. Until such approval is given, the president can only suspend the
assembly. In case the Parliament fails to approve the proclamation, the assembly would get
reactivated.

Q 88.B
• National Plan for Conservation of Aquatic Eco-systems (NPCA) is formed by merging the National Lake
Conservation Plan (NLCP) and National Wetland Conservation Programme (NWCP) in 2013.
• National Plan for Conservation of Aquatic Eco-systems (NPCA) is a centrally-sponsored scheme
implemented by the Ministry of Environment, Forest, and Climate Change. Hence statement 1 is
not correct.
• It deals with the conservation and management of lakes and wetlands in the country on a cost-
sharing basis between the central government and respective state/ UT governments. Hence,
statement 2 is correct.
• It covers various activities such as:
o interception, diversion, and treatment of wastewater
o shoreline protection
o lakefront development
o in-situ cleaning such as desilting & de-weeding
o fisheries development. Hence, statement 3 is correct.
o stormwater management bioremediation
o catchment area treatment
35 www.visionias.in ©Vision IAS

DEEPAK PHOTOSTAT 9310521834


o Lake Beautification
o survey & demarcation
o bio fencing
o weed control
o biodiversity conservation
o education and awareness creation
o community participation

Q 89.C
• WHAT IS COMPUTING? Computing is based upon a set of principles and ideas that work in a
hierarchical way ultimately giving way to some kind of technology depending on the process undertaken
to do the same. The technologies thus produced can be ever-evolving or constantly climbing the
complication ladder but the underlying ideas behind them remain the same.
• GRID COMPUTING? Grid computing works out perfectly for sorting or arranging large amounts of
data that is either bundled up wrong or needs to be spaced out.
o It works on the concept of many hundreds of interlinked and networked computers that work like
stacked-up software and handle the work like a supercomputer.
o It acts as a lender of efficient software computers that one can use for a specific amount of time, even
paying if necessary, to basically get rid of the otherwise tedious task of employing individual software
and front end – back end links for performing a given task.
o Grid computing offers a top tier method for people from a non-technological background to get their
work done with speed and precision. Since there are a number of computers involved in the
advancement, the run time of each task and therefore the time is taken in giving out the desired result
gets reduced a great deal to reduce wasted resources.
o Thus, the bottom line of grid computing is combining and deploying the efficiency of many hands to
reach the main objective. It is based on a processor grid computer that works on conjugated networks.
• TYPES OF GRID COMPUTING? The various types of grid computing are based solely on the task at
hand as well as the understanding of the user.
o Data grid – helps to handle, manipulate, discover and publish big masses of data that might be stored
in several heterogeneous systems
o Collaboration grid – helps in efficient combining of various data resources
o Network grid – is important for better working of software since it increases the fault tolerance of a
certain network and also helps to multiply the productivity of a given system
o Utility grid – gets its name from the fact that it not only provides a service for handling data and
calculation cycles but also the software involved. Centralized to and fro movement of information
being sent by the clients is an essential service that this type of grid computing provides.
o Information grid – used mainly for business apps and provides peer to peer servings
o Service grids – essentially provide a blend between the grid connections and the web-related
application involved. A perfect amalgam of the physical and software-based components.
• CLOUD COMPUTING? A cloud in the simplest terms is nothing but a network used for information
retrieval or for uploading the same. Serverless computing that involves the extraction of information by
the user from the internet through web-related searches without actually having to interfere with the
management of data is termed as cloud computing. It consists of an extensive IT infrastructure that serves
as a means for easy and competent retrieval of information from the internet.
• Collecting the manipulated, configured and accessed hardware and software components remotely are
basically termed as cloud computing.it is a more recent transition in the technological world but serves all
purposes with great precision. It is responsible for making the application more collaborative because it
offers platform independence, one does not need to physically install the software locally.
• TYPES OF CLOUD COMPUTING? The types of cloud computing are based on their deployment
services.
o Public cloud – made easily accessible for the general public but the security aspect is a little less
efficient because of the vast reach and openness of this type of cloud.
o Private cloud – generally works out best for an organization, business, or any integrated and
individual working environment. security is more tightened as compared to the public cloud.
o Community cloud – allows data to be retrieved by a number of similar organizations
o Hybrid cloud – a blend of private and public clouds in which the important activities are performed
on the private cloud and the others on the public cloud.
• GRID COMPUTING VS CLOUD COMPUTING? The key points of difference between cloud and
grid computing are: –
36 www.visionias.in ©Vision IAS

DEEPAK PHOTOSTAT 9310521834


o Architecture: –Whereas cloud computing a more client to server and vice versa computer
architecture, grid computing follows a more distributed form of architecture to meet the end objective.
o Resource: –Since cloud computing doesn’t have many computers involved in the same task, it
maintains the resources centrally in contrast to grid computing where the resources are
managed based on the collaboration pattern. Moreover, grid computing makes all the data
available in the virtual platform but cloud computing restricts the accessibility of resources to a
certain application directly. Hence statement 2 is correct.
o Functioning: –Grid computing works on the simple principle of dividing and distributing the
available resources into small tasks that every computer on the grid has to work with. When the
individual tasks are completed and arranged accordingly, the data goes back to the main machine.
Cloud computing however works on a need basis. Whenever there is a need for resources, the central
system distributes the resources and takes them back when the work is done.
o Flexibility: –cloud computing turns out to be more flexible in terms of its applications and efficiency
as compared to grid computing because cloud computing provides a wide array of information that
many people need but grid computing mostly works out for academic researchers since a vast amount
of data can be pooled and handled with utmost precision.
o Payment and set up: –This is one of the key differences between the two types of computing. Cloud
computing does not require any form of setting up of software whereas, in grid computing,
installation of the software component is essential and has to be done locally. Users pay for the
resource upon using the cloud in cloud computing but they need not pay any amount once the
initial set up is complete in grid computing. Hence statement 1 is correct.
o Accessibility: –Cloud computing renders more accessibility in comparison with grid computing but
mostly because of the difference in the end-users that use the two kinds of computing.

Q 90.B
• The Cool Temperate Eastern Margin (Laurentian) Climate:
o It is an intermediate type of climate between the British and the Siberian type of climate.
o It has features of both the maritime and the continental climates.
o The Laurentian type of climate is found only in two regions.
ü North American region: North-eastern North America, including eastern Canada, north-east
U.S.A., (i.e. Maritime Provinces and the New England states), and Newfoundland.
ü Asiatic region: Eastern coastlands of Asia, including eastern Siberia, North China, Manchuria,
Korea and northern Japan.
o In the southern hemisphere, this climatic type is absent because only a small section of the southern
continents extends south of the latitude of 40° S.
• Climatic Conditions:
o The Laurentian type of climate has cold, dry winters and warm, wet summers.
o Winter temperatures may be well below freezing-point and snowfalls to quite a depth.
o Summers are as warm as the tropics (21° - 27°C) and if it were not for the cooling effects of the off-
shore cold currents from the Arctic, the summer might be even hotter.
o Rain falls throughout the year, there is a distinct summer maximum from the easterly winds from the
oceans. Of the annual precipitation of 30 to 60 inches, two-thirds come in the summer.
o Winter is dry and cold because the winds are dry Westerlies that blowout from the continental
interiors.
o Hence, options (a), (c) and (d) are the characteristics of the Laurentian type of climate.
Moreover, the Warm Temperate Eastern Margin (China Type) climate is sometimes referred to as the
Gulf type of climate. Hence, option (b) is the correct answer.
• Natural Vegetation:
o The predominant vegetation of the Laurentian type of climate is cool temperate forest.
o The heavy rainfall, the warm summers and the damp air from fogs, all favour the growth of trees.
o The forest tends to be coniferous north of the 50° N. parallel of latitude. The increase in the length and
severity of the winter excludes forests that are not adaptable to cold conditions.
o Oak, beech, maple and birch are the principal trees.
• Economy
o Lumbering and its associated timber, paper and pulp industries are the most important economic
undertaking.
o Agriculture is less important in view of the severity of the winter and its long duration.
o The fertile Annapolis valley in Nova Scotia is the world’s most renowned region for apples.
o Fishing is the most outstanding economic activity of the Laurentian climatic regions.
37 www.visionias.in ©Vision IAS

DEEPAK PHOTOSTAT 9310521834


Q 91.D
• Seed germination is the growth process of a mature a seed, characterized by the emergence of a stem and
root from it is called as germination. Germination in plant refers to the process by which a seed begins to
sprout and grow into a seedling under the right growing conditions.
• Factors affecting seed germination:
o Internal factors: Permeability to water (Imperbeality of seeds to water prevents seed germination and
vice versa), Permeability to oxygen, development to embryo, after ripening processes, viability (It
refers the potential capacity of the seed to germinate).
o Size of seed: Bigger seeds generally give higher germination percentage than small seeds.
o External factors include Moisture, (Oxygen) air, temperature, light, seed bed etc. Seeds need
oxygen so that they can produce energy for germination and growth. The embryo gets energy by
breaking down its food stores. Like all organisms, this is done through a process known as aerobic
respiration. —a series of reactions where energy is released from glucose, using oxygen.
o Planting conditions and depth has direct impact on seed-to-soil contact as well as seeds' access to
adequate moisture and temperature. Planting too shallow may result in poor germination due to low
soil moisture retention near the soil surface or seed injury due to insects or disease.
• Hence option (d) is the correct answer.

Q 92.A
• Recently, an international team of astronomers have discovered the most distant ‘radio-loud’ quasar with
the help of European Southern Observatory’s Very Large Telescope (ESO’s VLT). It took 13 billion years
for the quasar’s light to reach earth. Quasars are very luminous astronomical objects in faraway
galaxies that emit jets at radio frequencies. They are only found in galaxies that have supermassive
blackholes which power these bright discs. However, 90 per cent of them do not emit strong radio
waves, making this newly-discovered one special. Hence statement 2 is not correct.
• A quasar is an extremely luminous active galactic nucleus (AGN), in which a supermassive black
hole with mass ranging from millions to billions of times the mass of the Sun is surrounded by a
gaseous accretion disk. Hence statement 1 is correct.
• Most active galaxies have a supermassive black hole at the centre which sucks in surrounding objects.
Quasars are formed by the energy emitted by materials spiralling around a blackhole right before being
sucked into it.
• Named P172+18, the quasar emitted wavelengths which had a redshift of 6.8. Only three other ‘radio-
loud’ sources with redshift greater than six have been discovered so far and the most distant one had a
redshift of 6.18. The higher the redshift of the radio wavelength, the farther away is the source.

Q 93.A
• Great Rann of Kutch Biosphere Reserve :
• The Great Rann of Kutch is a salt marsh in the Thar Desert in the Kutch District of Gujarat. The Great
Rann of Kutch, along with the Little Rann of Kutch and the Banni grasslands on its southern edge, is
situated in the district of Kutch. Hence option (d) is correct.
• The Luni River, which originates in Rajasthan, drains into the desert in the northeast corner of the
Rann. Other rivers feeding into the marsh include the Rupen from the east and the West Banas River
from the northeast.
• Great Rann of Kutch is famous for Indian Wild Ass. Hence option (a) is not correct.
• Tropic of Cancer Passes through it. Hence option (b) is correct.
• The Wild Ass Wildlife Sanctuary is spread in the entire area of the little Rann of Kutch where small
grass-covered areas, known as baits, forms the flora and is an important source to support the fauna of the
region. The key fauna of this desert is the Indian Wild Ass.
• The Luni River, which originates in Rajasthan, drains into the desert in the northeast corner of the Rann.
Other rivers feeding into the marsh include the Rupen from the east and the West Banas River from the
northeast. Hence option (c) is correct.

Q 94.C
• Afforestation of sand dunes: Afforestation refers to the establishment of the forest by artificial means on
an area from which forest vegetation has always or long been absent.
• Shifting sand dunes are distributed in Rajasthan, Gujarat, Haryana and Punjab and part of Tamil Nadu
(Their soils). These soils have low fertility due to poor structure, low clay content, low biological activity,
poor organic matter content and moisture.
• Afforestation measures: The afforestation work comprises of
38 www.visionias.in ©Vision IAS

DEEPAK PHOTOSTAT 9310521834


o Sand dune fixation and stabilization though micro-wind breaks,
o Creation of windbreaks and shelterbelts and
o Development of fuel and fodder blocks.
• Choice of species:
o The species selected for afforestation of sand dunes must be drought resistant and having a well-
developed root system. Hence statement 1 is correct.
o The tree should be able to penetrate deep into the soil for moisture and nutrient uptake.
o The root system should be multilayered i.e. the epidermis should be multilayered.
o The species should withstand high temperature and severe frost. The species should be a good copier
and should be able to provide firewood, posts, fodder, etc. Hence statement 2 is correct.

Q 95.D
• Forest Survey of India (FSI) is a premier national organization under the Union Ministry of Environment,
Forests, and Climate Change.
• It is responsible for the assessment and monitoring of the forest resources of the country on regular basis.
It is also engaged in providing training, research, and extension services in the forestry sector.
• Forest Survey of India releases the State of Forest Report biennially, providing an assessment of the
latest forest cover in the country and monitoring changes in these. Hence statement 1 is correct.
• In addition to forest cover, the tree cover of the country is also being carried out by FSI using the
Trees Outside Forests (TOF) inventory data. It measures tree cover outside forest areas such as
rural and urban areas which consist of orchards, etc. Hence statement 2 is correct.
• FSI has its headquarters at Dehradun and four zonal offices at Shimla (Northern Zone), Kolkata
(Eastern zone), Nagpur (Central Zone), and Bangalore (Southern zone). Recently, a sub-centre of the
Eastern zone has been established and made operational in Burnihat, Guwahati. Hence, statement 3 is
correct.
• As per the State of Forest Report, 2019 tree and forest cover together made up 24.56% (8,07,276 sq km)
of India's area. Of this, the tree cover is estimated at 95,027 sq km which is 2.89% of the geographical
area.
• Other major objectives and functions of FSI are
o To function as a nodal agency for collection, compilation, storage, and dissemination of spatial
database on forest resources.
o To support State/UT Forest Departments (SFD) in forest resources survey, mapping and inventory.
o To conduct training of forestry personnel in the application of technologies related to resources
survey, remote sensing, GIS, etc.
o To strengthen research & development infrastructure in FSI and to conduct research on applied forest
survey techniques.

Q 96.C
• The theremin is considered the world’s first electronic musical instrument and it turned 100 years old in
2020. It generates sound by modulation of electromagnetic waves and this is done without the player
having to even touch the instrument. It has a vertical antenna for controlling the tone and a looped antenna
for controlling the volume. Hence option (c) is the correct answer.
• It is named after its inventor Leon Theremin, who invented it accidentally while experimenting with radio
waves. The invention of the instrument revolutionized music. It has its history tied to the prisons of
Siberia, world war and Hollywood films of 20th century.
• It has a vertical antenna and a loop antenna. The musicians modulate the electromagnetic field in the
instrument by moving their hands and fingers around the space. When the musician move around the
vertical antenna he can increase or decrease the tone. When he moves around the loop antenna, he can
control the volume.
• The AlphaSphere is a spherical instrument that consists of 48 tactile pads that respond to pressure as well
as touch. Custom software allows the pads to be indefinitely programmed individually or by groups in
terms of function, note, and pressure parameter among many other settings. The primary concept of the
AlphaSphere is to increase the level of expression available to electronic musicians, by allowing for the
playing style of a musical instrument.
• The Kaossilator and Kaossilator Pro are compact instruments where the position of a finger on the
touchpad controls two note-characteristics; usually, the pitch is changed with a left-right motion and the
tonal property, filter or other parameter changes with an up-down motion. The touchpad can be set to
different musical scales and keys. The instrument can record a repeating loop of adjustable length, set to
any tempo, and new loops of sound can be layered on top of existing ones. This lends itself to electronic
39 www.visionias.in ©Vision IAS

DEEPAK PHOTOSTAT 9310521834


dance-music but is more limited for controlled sequences of notes, as the pad on a regular Kaossilator is
featureless.
• The Reactable is a round translucent table with a backlit interactive display. By placing and manipulating
blocks called tangibles on the table surface, while interacting with the visual display via finger gestures, a
virtual modular synthesizer is operated, creating music or sound effects. It is a round translucent table,
used in a darkened room, and appears as a backlit display. The table itself is the display. As a tangible is
placed on the table, various animated symbols appear, such as waveforms, circles, circular grids, or
sweeping lines. Some symbols merely show what the particular tangible is doing, others can be used by
fingertip to control the respective module.

Q 97.C

• Nations are described as ‘federal’ or ‘unitary’, depending on the way in which governance is organised.
Federalism essentially means both the Centre and states have the freedom to operate in their allotted
spheres of power, in coordination with each other.In the unitary system all powers of the Government are
centralized in one Government that is Central Government.
• Federalism is a system of government in which power is divided between a central authority and
constituent political units. The Constitution of India establishes a federal structure to the Indian
government, declaring it to be a "Union of States".
• In S R Bommai vs Union of India (1994), Supreme Court held federalism a part of the basic structure of
the Constitution.
• Differences between Federalism in India and USA
o In India, seats are allotted to states in Rajya Sabha (Upper House) on the basis of Population. Hence,
different states have different seats in Rajya Sabha. On the other hand, in USA, all states are given
equal representation in the Senate irrespective of their population. USA has 50 states and the Senate
has 100 members – 2 from each states. Hence option (a) is not correct.
o In India, the power to make laws with respect to residuary subjects (i.e. the matters which are not
enumerated in any of three lists) is vested in the Parliament. In USA, residuary powers are given to
the states. Hence, option (b) is not correct.
o Indian Parliament has the power to admit new States (Article 2), create new States, alter their
boundaries and their names, and unite or divide the States (Article 3). For example - Recently, Jammu
and Kashmir (J&K) State was converted into two Union Territories - J&K and Ladakh. The
concurrence of States is not needed for the formation and unmaking of States and Union Territories.
Thus states in India are not indestructible. Hence option (d) is not correct.
• Indian Federation is described as an indestructible Union of destructible states. It means that states have
no right to secede from the federation. The federation is union because it is indestructible. On the other
hand, American Federation is described as an indestructible Union of indestructible states. Here also, the
Union is indestructible. Hence option (c) is correct.

Q 98.A
• Solifluction: Solifluction is a mass movements process, which involves slow downslope flowing soil
mass or fine-grained rock debris saturated or lubricated with water. Hence, statement 1 is correct.
o This process is quite common in moist temperate areas where surface melting of deeply frozen ground
and long-continued rain respectively, occur frequently. When the upper portion gets saturated and
when the lower parts are impervious to water percolation, flowing occurs in the upper parts. Hence,
statement 2 is not correct.
• Mass movements: Mass movements transfer the mass of rock debris down the slopes under the direct
influence of gravity. That means, air, water or ice do not carry debris with them from place to place but on
the other hand, the debris may carry with it air, water or ice.
o Mass movements are aided by gravity and no geomorphic agent like running water, glaciers, wind,
waves and currents participate in the process of mass movements.
o It can be grouped under two major classes:
• Slow movements
o Creep- the extremely slow movement of material, imperceptible except through extended observation.
o Solifluction
• Rapid movement
o Earth flow
ü Mudflow
ü Debris Avalanche
40 www.visionias.in ©Vision IAS

DEEPAK PHOTOSTAT 9310521834


ü Landslides
ü Slump
ü Debris slide
ü Rockslide
Q 99.C
• The privy purse was a monetary benefit provided to erstwhile princely states in return of their sovereignty
claim. It was abolished in 1971 as a 26th constitutional amendment. Then Prime Minister Indira Gandhi
argued the case for abolition based on equal rights for all citizens and the need to reduce the government's
revenue deficit. Article 363-A was inserted in the Constitution (Twenty-sixth Amendment) Act, 1971 for
the abolition of Privy Purse.
• The 26th Constitutional Amendment Act 1971, which was moved by the then Prime Minister Indira
Gandhi, abolished abolished privy purses and privileges of the Rulers of former Indian States. Hence
option (c) is the correct answer.
• 26th Constitutional Amendment Act 1971
o It derecognized the titles such as 'Ruler' and also abolished the privy purse and all rights, liabilities
and obligations in respect of privy purse.
o The amendment made four changes to the then existing Constitution. It deleted Articles 291 and 362
of the Constitution, inserted a new Article 363A and also amended Article 366.
o Article 291 dealt with payment of Privy Purse sums of Rulers
o Article 362 dealt with rights and Privileges of rulers of Indian States. Article 362 mandated the
executive and legislature to give due regard to the guarantee or assurance given under any such
covenant or agreement as is referred to in Article 291 with respect to the personal rights, privileges
and dignities of the Ruler of an Indian State. Both, Articles 291 and 362 ceased to exist after 26th
Amendment.
o The 26th Amendment also inserted Article 363A to terminate expressly the recognition already
granted to such Rulers and to abolish privy purses and extinguish all rights, liabilities and obligations
in respect of privy purses.
• SC Rejected Challenge Against 26th Amendment
o The constitutional validity of 26th Amendment Act was challenged in SC in Raghunathrao Ganpatrao
Etc. Etc vs Union Of India case. Sri Raghunathrao Raja, who was the Co-Ruler of Indian State of
Kurundwad, challenged the constitutional validity of the Constitution (Twenty-sixth Amendment) Act
of 1971 on the ground that it violates the basic structure and essential features of the Constitution of
India and is, therefore, outside the scope and ambit of constituent powers of the Parliament to amend
the Constitution as provided under Article 368 of the Constitution.
o The Court, while upholding the 26th Constitutional Amendment, held that the Amendment did not
offend Article 14 or 19(g). It further held that no principle of justice, either economic, political or
social is violated by the Twenty-sixth Amendment.
Q 100.A
• Article 17 abolishes ‘untouchability’ and forbids its practice in any form. However, untouchability
is not defined in the Constitution. Hence statement 1 is correct.
• In order to restrict the practise of untouchability, Parliament enacted Under the Protection of Civil
Rights Act, 1955 which was amended in 1976 recognises practise of ‘untouchability’ as non-bailable
and cognizable offence. Hence statement 3 is correct.
• It is the Representation of People Act,1951 which was enacted by the parliament, recognises practise of
‘untouchability’ as a ground for disqualification of member of Parliament. Hence statement 2 is
not correct.
• Additional information: Under the Constitution, a person shall be disqualified for being elected as a
member of Parliament:
o if he holds any office of profit under the Union or state government (except that of a minister or
any other office exempted by Parliament).
o if he is of unsound mind and stands so declared by a court.
o if he is an undischarged insolvent.
o if he is not a citizen of India or has voluntarily acquired the citizenship of a foreign state or is under
any acknowledgement of allegiance to a foreign state; and
o if he is so disqualified under any law made by Parliament.

Copyright © by Vision IAS


All rights are reserved. No part of this document may be reproduced, stored in a retrieval system or transmitted
in any form or by any means, electronic, mechanical, photocopying, recording or otherwise, without prior
41permission of Vision IAS. www.visionias.in ©Vision IAS

DEEPAK PHOTOSTAT 9310521834

You might also like